Neural sciences Flashcards

1
Q

A 64-year-old lady, on physical examination exhibits symptoms suggestive
of a movement disorder with associated speech defi cits. This clinical
presentation is classifi ed as ‘hypokinetic dysarthria’ by her neurologist. It is
associated with
A. Parkinson’s disease
B. Huntington’s disease
C. Spasmodic dysphonia
D. Multiple sclerosis
E. Myasthenia gravis

A

The answer is Parkinson’s disease. Bradykinesia or hypokinesia is a motor feature of
Parkinson’s disease. Dysarthria is a defi cit in the motor aspect of speech. It is usually secondary
to a motor neurological defi cit. Dysarthria can affect not only articulation, but also phonation,
breathing, or prosody (emotional tone) of speech. Total loss of ability to articulate is called
anarthria, whereas dysarthria usually involves the distortion of consonant sounds. The Mayo
Clinic classifi cation of dysarthria divides dysarthria into six basic types, each one corresponding
to a predominant motor disorder: fl accid (lower motor neurone disorders), spastic (upper
motor neurone disorders), ataxic (cerebellar lesions), hypokinetic (parkinsonian), hyperkinetic
(choreiform/tic disorders), and mixed. Mixed dysarthrias are seen in conditions with multiple
motor lesions, for example mixed spastic–ataxia of multiple sclerosis or mixed spastic–fl accidity
of amyotrophic lateral sclerosis. Speech therapy may be of substantial benefi t to many dysarthric
patients.

How well did you know this?
1
Not at all
2
3
4
5
Perfectly
2
Q

A 32-year-old man is diagnosed with a right-sided hemiparesis. On
examination, his speech shows non-fluent aphasia. His comprehension is
intact, but repetition is impaired. He is most likely to have
A. Transcortical motor aphasia
B. Transcortical sensory aphasia
C. Conduction aphasia
D. Broca’s aphasia
E. Wernicke’s aphasia

A

D. Testing a person’s speech is usually done in three steps. The fi rst step is to test for the
fl uency of speech. Non-fl uent output is characterized by a paucity of verbal output (usually
10–50 words per minute), whereas fl uent aphasics have a normal or even exaggerated verbal
output (up to 200 words or more per minute). Lesions of the motor (Broca’s) area produce
a non-fl uent aphasia. Assessment of language comprehension is the second step. Patients with
focal lesions limited to the left frontal lobe (Broca’s area) will have preserved comprehension
(Broca’s and transcortical motor aphasia). Patients with left posterior temporal or parietal
involvement will show impaired comprehension (Wernicke’s, global, transcortical sensory, and
isolation aphasias). The third step is to evaluate repetition. Transcortical aphasias usually have an
intact repetition. Patients with Broca’s, Wernicke’s, or conduction aphasia typically show impaired
repetition. In conduction aphasia, speech is fl uent (as in Wernicke’s aphasia) but comprehension is
intact (unlike Wernicke’s aphasia

How well did you know this?
1
Not at all
2
3
4
5
Perfectly
3
Q
A patient presents with features suggestive of Gerstmann’s syndrome. He
also has aphasia. Which of the following is the most likely type of aphasia
with which he may present?
A. Transcortical sensory aphasia
B. Transcortical motor aphasia
C. Anomic aphasia
D. Global aphasia
E. Broca’s aphasia
A

A. Transcortical sensory aphasia is similar to Wernicke,s aphasia but is distinguished by the
retained ability to repeat. Lesions causing transcortical aphasias do not disrupt the perisylvian
language circuit from Wernicke’s area through the arcuate fasciculus to Broca’s area. Instead, they
interrupt connections from other cortical centres into the language circuit (hence the name
transcortical). These areas include the dominant angular gyrus, posterior middle temporal gyrus,
and periventricular white matter pathways of the temporal isthmus underlying these cortical
areas. When this results from involvement of the angular gyrus, it is frequently accompanied
by Gerstmann’s syndrome, constructional apraxia, and other evidence of the angular gyrus
syndrome.

How well did you know this?
1
Not at all
2
3
4
5
Perfectly
4
Q

Regarding aphasia, which of the following statements is true?
A. Broca’s aphasia presents with logorrhoea.
B. Neologism is a feature of Broca’s aphasia.
C. Paragrammatism is a feature of Wernicke’s aphasia.
D. Pure word deafness is associated with loss of naming.
E. Involvement of the posterior cerebral artery leads to global aphasia

A

C. Paragrammatism is seen in Wernicke’s aphasia. Speech is characterized by being empty
of meaning, containing verbal paraphasias, neologisms, and jargon productions. Most patients
with Wernicke’s aphasia have no elementary motor or sensory defi cits. A right homonymous
hemianopia may be present. Patients may be unaware of the defi cit and may present with
paranoia, as they do not realize why others do not understand them. The presence of
paragrammatism may be diffi cult to distinguish from formal thought disorder in schizophrenia.
In contrast, Broca’s aphasia shows agrammatism. In this case, the speech pattern is non-fl uent;
on examination, the patient speaks hesitantly, often producing the principal, meaning-containing
nouns and verbs but omitting small grammatical words and morphemes. This pattern is called
agrammatism or telegraphic speech, for example: ‘I go home’ or ‘wife here morning’. Reading is
often impaired in Broca’s aphasia despite preserved auditory comprehension. Broca’s aphasia
is associated with right hemiparesis, hemisensory loss, and apraxia of the non-paralysed left
limbs. Due to the awareness of the defi cit, patients with Broca’s aphasia may be more prone to
depression. Pure word deafness is a syndrome of isolated loss of auditory comprehension and
repetition, without any abnormality of speech, naming, reading, or writing. It is caused by bilateral,
or sometimes a unilateral, lesion, isolating Wernicke’s area from input from both Heschl’s gyri.
A lesion representing most of the territory of the left middle cerebral (not posterior circulation)
artery leads to a global aphasia.

How well did you know this?
1
Not at all
2
3
4
5
Perfectly
5
Q

Which of the following is true regarding acquired defects in reading
and writing?
A. Alexia without agraphia is called acquired illiteracy.
B. Alexia without agraphia is seen in association with Gerstmann’s syndrome.
C. Anomic aphasia is associated with Gerstmann’s syndrome.
D. Transcortical aphasia is due to lesions in the arcuate fasciculus.
E. Alexia without agraphia is seen in posterior cerebral artery stroke.

A

E. Pure alexia without agraphia is associated with left posterior cerebral artery stroke,
with infarction of the medial occipital lobe, often the splenium of the corpus callosum, and often
the medial temporal lobe. Alexia is the acquired inability to read. Patients with alexia without
agraphia can write but cannot read their own writing. Alexia with agraphia is sometimes called
acquired illiteracy. Alexia with agraphia is seen in angular gyrus lesions and is associated with
Gerstmann’s syndrome. It is seen in stroke of the angular branch of the middle cerebral artery.
Transcortical aphasias are analogues to the syndromes of global, Broca’s, and Wernicke’s aphasias,
with intact repetition. Lesions producing transcortical aphasias disrupt connections from other
cortical centres into the language circuit. Lesions to the arcuate fasciculus (usually in either the
superior temporal or inferior parietal regions) present with conduction aphasia

How well did you know this?
1
Not at all
2
3
4
5
Perfectly
6
Q
The clinical sign of fi nger–nose ataxia is seen in lesions of which of the
following structures?
A. Superior colliculus
B. Inferior colliculus
C. Pyramidal decussation
D. Inferior olivary nucleus
E. Thalamus
A

D. Inferior olivary lesions lead to appendicular ataxia which can be tested using the fi nger–
nose test. The inferior olivary nucleus serves motor coordination via projecting climbing fi bres
to the cerebellum. Isolated lesions of superior colliculus result in defective visual saccades. Subtle
auditory defects are noted in similar lesions of the inferior colliculus. Pyramidal decussation
carries corticospinal fi bres; damage to the corticospinal fi bres rostral to (above) the pyramidal
decussation results in contralateral motor defi cits, while lesions below the decussation result in
ipsilateral defi cits. Thalamic damage often results in sensory defi cit syndromes

How well did you know this?
1
Not at all
2
3
4
5
Perfectly
7
Q

A patient is observed to be repeating the phrases or words spoken by the
examiner. Which of the following can cause this phenomenon?
A. Transcortical motor aphasia
B. Transcortical sensory aphasia
C. Mixed transcortical aphasia
D. Huntington’s disease
E. All of the above

A

E. Echolalia is the phenomenon where the patient repeats words or phrases said by the
examiner; palilalia is the phenomenon where the patient repeats words or phrases that he
has uttered himself. In patients who develop both phenomena, echolalia precedes the onset of
palilalia. Common causes of echolalia include the transcortical aphasias and disorders that affect
the basal ganglia–frontal circuit. Echolalia could be due to a frontal executive defi cit, leading to
failure of environmental autonomy and resulting in echoing of perceived environmental stimuli.
Palilalia should be distinguished from stuttering and logoclonia (repetition of the fi nal syllable of
spoken words). Echolalia may be observed as part of speech disturbances in catatonic states

How well did you know this?
1
Not at all
2
3
4
5
Perfectly
8
Q

Neuropsychiatric Interview (NPI) is often employed in patients with
dementia or cognitive deterioration to detect psychiatric and behavioural
problems. Which of the following is not tested by the NPI?
A. Thought disturbance
B. Perceptual disturbance
C. Affective disturbance
D. Abnormalities in sleep pattern
E. Disorientation

A

E. Orientation is a measure of cognitive function. NPI is used for the assessment of thought
disturbance, perceptual disturbances, affect, abulia, agitation/aggression, disinhibition, appetite
disturbance, sleeping pattern, and aberrant motor activity in patients with dementia/cognitive
defi cits. It does not test cognitive functions such as memory or orientation

How well did you know this?
1
Not at all
2
3
4
5
Perfectly
9
Q

Regarding handedness, which of these statements is true?
A. The population can be divided into two categories: right and left handed.
B. 60% of the population are right handed.
C. 75% of right-handed people are left-hemisphere dominant for language.
D. 60% of left-handed people are left-hemisphere dominant for language.
E. Left-handed people are less likely than right-handed ones to have bilateral language
representation.

A

D. Hemispheric dominance is clinically inferred by handedness. It is a peripheral indicator
of cerebral hemispheric language lateralization. Handedness is now considered to exist as a
continuum, from extreme unilateral hand dominance on one end to ambidexterity on the other.
In this respect, the Edinburgh Handedness Inventory is a semiquantitative measurement of
handedness. It is thought that at least 90% of the human population is right-handed. Of these,
95% are left-hemisphere dominant. Approximately 10% of the human population is left-handed
and of these at least 60% are left-hemisphere dominant. Left-handers are more likely to have
bilateral language representation.

How well did you know this?
1
Not at all
2
3
4
5
Perfectly
10
Q

A patient with a history of traumatic brain injury undergoes
neuropsychological testing. In part A of the test he is asked to connect
numbered circles on a paper as fast as he can in correct order, using a pen.
In part B of the same test the same task is repeated but numbers and
alphabets occur in alternate sequences. Which of the following statements
is correct with regard to this test?
A. This is called letter cancellation task.
B. This test is not sensitive to progressive cognitive decline in dementia.
C. Part A of the test corresponds more closely to executive functioning than part B.
D. Patients with traumatic injury perform this test slower than average.
E. This is purely a test of selective attention.

A

D. This test is called the trail making test. It is not only a test of attention, but it also tests
visuomotor tracking and cognitive fl exibility (part B). Trail making test A requires the subject to
connect numbered dots. Trail making test B requires the subject to connect alternating alphabets
and numbers. This tests the ability to shift mental sets and hence to some extent corresponds to
executive functioning. This has been shown to be sensitive to change in patients with progressive
cognitive decline (e.g. dementia). Patients with traumatic brain injury perform slower on trail
making tests.

How well did you know this?
1
Not at all
2
3
4
5
Perfectly
11
Q

Which of the following matches is incorrect regarding amnestic syndrome
and site of lesion?
A. Wernicke–Korsakoff syndrome–thalamic nuclei
B. Herpes simplex encephalitis–anterior temporal cortex
C. Crutzfeld–Jakob disease–diffuse cortical
D. Anterior communicating artery stroke–medial temporal cortex
E. Complex partial seizures–hippocampal damage

A

D. Medial temporal cortex is not supplied by the anterior communicating artery; it is
supplied by the posterior cerebral artery. The anterior communicating artery supplies the basal
forebrain and striatum. Wernicke–Korsakoff syndrome is usually associated with nutritional
causes, where the thalamic nuclei (especially dorsal medial thalamus) are involved, leading to
anterograde amnesia and confabulation. Herpes simplex encephalitis (HSE) is another cause
of anterograde amnesia where anterior temporal lobes are often involved. Whether amnesia
is predominantly verbal or non-verbal is determined by the side of lesion and the cerebral
dominance. In CJD (Creutzfeldt–Jacob disease) diffuse cortical damage occurs. Amnesia of
complex partial seizures is related to recurrent hippocampal damage and sclerosis

How well did you know this?
1
Not at all
2
3
4
5
Perfectly
12
Q
Which of the following is a component of the triad in Balint’s syndrome?
A. Visual neglect
B. Achromatopsia
C. Prosopagnosia
D. Simultanagnosia
E. Anosognosia
A

D. Balint’s syndrome consist of a triad of oculomotor apraxia (defi cits in the orderly
visuomotor scanning of the environment), optic ataxia (inaccurate manual reaching toward visual
targets), and simultanagnosia. Pathologically, Balint’s syndrome is produced by bilateral parietooccipital
lesions. Simultanagnosia is the inability to integrate visual information in the centre of
gaze with more peripheral information. The patient gets stuck on the detail that falls in the centre
of gaze without scanning the visual environment for additional information. They typically ‘miss
the forest for the trees.’ This leads to a signifi cant disturbance in object identifi cation. A patient
with Balint’s syndrome when shown a table lamp and asked to name the object may look at its
circular base and call it an ash tray!

How well did you know this?
1
Not at all
2
3
4
5
Perfectly
13
Q

Features of Gerstmann’s syndrome include all of the following except
A. Dysgraphia
B. Finger agnosia
C. Dysarthria
D. Inability to distinguish left from right
E. Acalculia

A

C. Dysarthria is not a feature of Gerstmann’s syndrome. Full Gerstmann’s syndrome, though
rarely reported, consists of left–right disorientation, fi nger agnosia, dysgraphia, and dyscalculia.
The lesion is mostly attributed to a dominant parietal lobe dysfunction. When all the components
are present the syndrome reliably localizes to the dominant angular gyrus. Gerstmann himself
thought that the inability to calculate was because of the fact that children learnt to count with
their fi ngers and the dysgraphia was due to problems with differential fi nger movements—both
being secondary to fi nger agnosia. Gerstmann noted that the greatest trouble in fi nger agnostics
was with distinguishing second, third, and fourth fi ngers. Screening for full Gerstmann’s syndrome
should be performed on patients who show any single component.

How well did you know this?
1
Not at all
2
3
4
5
Perfectly
14
Q
Blindsight is a feature of which of the following focal cortical syndromes?
A. Balint’s syndrome
B. Geschwind’s syndrome
C. Charcot–Willibrand syndrome
D. Anton’s syndrome
E. Central achromatopsia
A

D. Anton’s syndrome features blindness and denial of blindness, that is the patient is blind
but denies sightlessness. The syndrome is most commonly associated with bilateral lesions of the
occipital cortex. Blind sight is a paradoxical syndrome seen in patients with cortical blindness.
It is the ability of the person to orient towards visual stimuli while there is no conscious visual
perception. This is due to the fact that 20 to 30% of fi bres of the optic tract are directed to
non-geniculate destinations, such as the superior colliculi and pretectal region of the brainstem.
It is thought that some visual processing occurs in this non-geniculate system. This phenomenon
is not demonstrable if the blindness is the result of pregeniculate lesions. Geschwind’s syndrome
refers to personality changes proposed to be due to disconnection of brain areas noted in
those with temporal lobe epilepsy. The Charcot–Wilbrand syndrome, or irreminiscence, is
characterized by the inability to generate an internal mental image or revisualize (imagine) an
object. The patients have more diffi culty in generating objects through drawing than in copying
model fi gures. It is usually secondary to bilateral parietal lobe lesions. Central achromatopsia
refers to loss of colour vision due to occipital lobe lesions.

How well did you know this?
1
Not at all
2
3
4
5
Perfectly
15
Q
A patient is not able to perform sequential motor acts despite intact
comprehension, muscle power, and ability to perform single-step commands.
He is exhibiting
A. Ideational apraxia
B. Ideomotor apraxia
C. Conceptual apraxia
D. Conduction apraxia
E. Dissociation apraxia
A

A. Ideational apraxia (IDA) is an inability to correctly sequence a series of acts that lead to
a goal. Asking the patient to carry out a multistep, sequential task, such as preparing a sandwich for
work, is a good test of IDA. It is most often associated with degenerative dementia and delirium.
Ideomotor apraxia is probably the most common type of apraxia. Patients with ideomotor
apraxia make spatial and temporal errors when performing learned, skilled movements including
pantomimes, imitations, and using actual objects. When pantomiming the use of a screwdriver,
patients with ideomotor apraxia may rotate their arm at the shoulder and fi x their elbow. In
right-handed individuals ideomotor apraxia is almost always associated with left-hemisphere
lesions. A variety of structures, including the corpus callosum, the inferior parietal lobe, and the
premotor areas, may be involved. Patients with ideomotor apraxia can imitate actions of others
(using tools/objects) but have diffi culty pantomiming (in the absence of tools/objects). In patients
with conduction apraxia, imitation is worse than pantomiming. The site of the lesion has not been
localized (unlike conduction aphasia). Patients with conceptual apraxia make tool-selection errors.

How well did you know this?
1
Not at all
2
3
4
5
Perfectly
16
Q

Which of the following is true about limb-kinetic apraxia?
A. Tasks such as fi nger tapping and pegboard are typically unimpaired.
B. Picking up objects using pincer grasp is spared.
C. It usually affects the hand that is ipsilateral to a hemispheric lesion.
D. The lesion is localized to the contralateral premotor cortex.
E. Patients typically present with an inability to perform multistep motor task

A

D. Limb-kinetic apraxia most often occurs in the limb contralateral to a hemispheric lesion,
usually to the premotor cortex. Patients with limb-kinetic apraxia demonstrate a loss of deftness
and ability to make fi nely graded, precise, independent fi nger movements. These subjects will not
be able to use a pincher grasp to pick up a penny. They will have trouble rotating a coin between
the thumb, middle fi nger, and little fi nger.

How well did you know this?
1
Not at all
2
3
4
5
Perfectly
17
Q
A 55-year-old man fi nds it diffi cult to recognize faces. On further testing,
his ability to discriminate faces and match faces is intact. The most likely
condition he is suffering from is
A. Apperceptive prosopagnosia
B. Associative prosopagnosia
C. Apperceptive visual object agnosia
D. Simultanagnosia
E. Central achromatopsia
A

B. The term agnosia was originally introduced by Freud. In general, patients with agnosia
have clinical feature of impaired recognition of sensory stimuli despite normal sensory pathways.
Agnosia represents a disorder of higher-order sensory processing. There is an impaired ability
to recognize the nature or meaning of sensory stimuli. This is usually modality specifi c. There
are two basic categories of agnosia. Apperceptive agnosia involves impaired generation of
the minimal integrated percept necessary for meaningful recognition. This defect, leads to
the formation of an inadequate minimal object recognition unit (i.e. the minimum information
required to meaningfully interpret the percept). For example, a “pencil” is initially perceived
as—”long, thin, pointed at one end, etc”, before a meaning (“it is a pencil—it is used to write”)
is attributed to the percept. Patients are unable to distinguish visual shapes and so have trouble
recognizing, copying, or discriminating between different visual stimuli. Associative agnosia involves
defective association of meaning with percepts. The defect is in associating a correctly perceived
percept with its meaning. Patients can describe visual scenes and classes of objects but still fail to
recognize them. Patients suffering from associative agnosia are still able to reproduce an image
through copying. Anosognosia refers to being unaware of a neurological state/illness. Abulia refers
to loss of drive or motivation seen in cingulate lesions.

How well did you know this?
1
Not at all
2
3
4
5
Perfectly
18
Q

Which of the following is true regarding episodic memory?
A. Episodic memory is implicit and non-declarative.
B. Episodic memory loss is not seen without medial temporal lesions.
C. Episodic memory loss can present as anterograde or retrograde amnesia.
D. Episodic memory applies only to events of personal signifi cance.
E. Episodic memory is more often preserved than semantic memory in dementia

A

C. Memories of specifi c experiences formed in specifi c contexts are called episodic, for
example the meal one had 3 weeks ago at a restaurant. Episodic memory is explicit, that is it is
consciously acquired (we know how and where we acquire it) and declarative, that is it can be
consciously recalled. Episodic memory depends largely on the integrity of the medial temporal
lobe, but there are other structures that are involved in episodic memory. These include the
frontal lobe, basal forebrain, retrosplenial cortex, presubiculum, fornix, mammillary bodies,
mammillothalamic tract, anterior nucleus of the thalamus, etc. Damage to any one of these
structures can result in defi cits in episodic memory. Hence episodic memory loss cannot be said
to be characteristic of a medial temporal lesion. Episodic memory impairment could manifest as
anterograde or retrograde amnesia. Anterograde amnesia refers to impairment in new memory
formation and retrograde amnesia refers to the loss of previously acquired memories. Episodic
memory applies to both personal and public events. In most dementias, semantic memory loss
occurs at later stages than episodic memory loss.

How well did you know this?
1
Not at all
2
3
4
5
Perfectly
19
Q
During bedside cognitive testing, a 40-year-old patient is asked to give the
years when World War II took place. Which of the following memories is
tested here?
A. Procedural memory
B. Episodic memory
C. Semantic memory
D. Implicit memory
E. Non-declarative memory
A

C. Semantic memory describes memories for general information which is unrelated
to other information, for example dates in history, the colour of our national fl ag, or the
characteristics of different species of dinosaurs (encyclopaedic facts). Semantic memory is explicit
and declarative (see explanation to the previous question). In the most general sense, semantic
memory refers to all of our knowledge of the world; however, semantic memory is more usually
tested in the context of naming and categorization tasks. It is localized to the inferior lateral
temporal lobes. The frontal lobes are responsible for providing information to, and retrieving
information out of, the semantic memory banks

How well did you know this?
1
Not at all
2
3
4
5
Perfectly
20
Q

A 35-year-old woman recently separated from her boyfriend of 5 years was
brought to the A&E with loss of memory. On examination, her memory
loss is specifi c to events associated with her boy friend. But she remembers
other events that took place around the same time. She is most probably
suffering from
A. Localized amnesia
B. Selective amnesia
C. Generalized amnesia
D. Continuous amnesia
E. Systematized amnesia

A

E. The woman described in the question probably suffers from dissociative amnesia, in
this case precipitated by the stress of separation. Systematized amnesia is the loss of memory
for a certain category of information such as material relating to one’s family or a particular
person. In this case, her boy friend. Localized amnesia is the condition where the individual fails
to recall events that occurred during a circumscribed period of time. In selective amnesia the
person can recall some but not all events during a circumscribed period of time. Generalized
amnesia is characterized by a failure to recall all of a person’s past life. There may be dissociation
between explicit and implicit memory, for example the person may retain all his learned skills,
but completely forget who he is or his past (a la Jason Bourne in the Bourne trilogy). Continuous
amnesia is a condition featuring an inability to recall events subsequent to a specifi c time up to
and including the present.
Cummings JL and Mega MS

How well did you know this?
1
Not at all
2
3
4
5
Perfectly
21
Q

After an enjoyable evening with friends at a pub, Tom calculates the cost
of the number of drinks that he had, subtracts the total from the value of
money he gave the bartender, and calculates the change that is due. The
system of memory that enables such calculation is
A. Episodic memory
B. Semantic memory
C. Procedural memory
D. Working memory
E. Retrograde memory

A

D. Working memory describes the ability to temporarily hold information in mind
and manipulate it as required by circumstances, for example doing mental arithmetic. It may
be phonological, such as keeping a phone number in mind for as long as it takes to dial or
visuospatial, such as following a mental map while cycling to work. Baddeley described a central
executive system in working memory, which is central to manipulation of the data held in the
‘phonological loop’ or the ‘visuospatial sketchpad’. In short, working memory is what allows us
to mentally add up the cost of the number of pints of lager we had at the pub, subtract the total
from the value of the money we give the bar tender, and calculate the change that is due to us.
Prefrontal cortex is the most important structure for working memory, due to the extensive
role played by the central executive; other structures involved include posterior parietal cortices.
Disturbances of working memory can result in anterograde disturbances to other systems
of memory as well, because intact working memory is generally required for the encoding of
information. Episodic memory may be particularly affected

How well did you know this?
1
Not at all
2
3
4
5
Perfectly
22
Q
Which of the following conditions does not show predominant abnormality
in procedural memory?
A. Parkinson’s disease
B. Huntington’s disease
C. Progressive supranuclear palsy
D. Olivopontocerebellar degeneration
E. Early Alzheimer’s disease
A

E. Procedural memory describes the ability to learn and perform tasks without
conscious thought. This is disturbed in conditions that involve subcortical basal ganglia
structures such as Parkinson’s disease, Huntington’s disease, progressive supranuclear palsy,
and olivopontocerebellar degeneration. Procedural memory defi cits may also be found in
depression and OCD. In conditions such as Alzheimer’s disease, mild cognitive impairment, Lewy
body dementia, vascular dementia, the frontal variant of frontotemporal dementia, encephalitis,
Korsakoff ’s syndrome, traumatic brain injury, hypoxic–ischaemic brain injury (including cardiac
bypass surgeries), temporal lobe surgery, seizures, vitamin B12 defi ciency, hypoglycaemia, transient
global amnesia, and multiple sclerosis, episodic memory is more likely to be impaired. Mood,
anxiety, and psychotic disorders may also show episodic memory disturbances. Finally, episodic
memory impairment may be a side-effect of treatment with anticholinergic drugs and ECT.
Semantic memory may be disturbed in conditions such as Alzheimer’s disease, the temporal
variant of frontotemporal dementia, traumatic brain injury, and encephalitis. Working memory
is disturbed in most of the conditions listed above. Working memory is also impaired in anxiety,
depression, schizophrenia, OCD, ADHD, other psychiatric states, and medications. Finally,
impairments in working memory occur as part of normal ageing.

How well did you know this?
1
Not at all
2
3
4
5
Perfectly
23
Q

A patient who had developed a pyloric stenosis following ingestion of
sulphuric acid develops a confusional state, ophthalmoplegia, and ataxia.
Which of the following is not true?
A. A CT scan may reveal bilateral hypodense areas in the medial thalamus.
B. The patient may present with diffi culty in learning new information.
C. Administration of thiamine in the acute phase may prevent the emergence of chronic
amnesic syndrome.
D. Confabulation is most common in the early stage of the amnesic syndrome.
E. The patient’s memory of events before the onset of amnesia is always normal.

A

E. The condition described is Wernicke–Korsakoff syndrome. Although the common
cause for the syndrome is malnutrition secondary to alcohol use, a number of other conditions
including hyperemesis during pregnancy, gastrectomy, pyloric stenosis, etc. are associated. In
addition to diffi culty learning new information, patients with Korsakoff ’s syndrome usually have
a retrograde amnesia which could extend back up to several years prior to the onset of the
syndrome. Patients usually remain amnesic for 1–3 months after onset and then begin to recover
gradually over a 10-month period; 25% recover completely and 25% have no demonstrable
recovery. CT scan may reveal bilateral hypodense areas in the medial thalamus in patients
with acute Wernicke’s encephalopathy, and mamillary body atrophy may be demonstrated by
MRI in some patients with chronic Korsakoff ’s syndrome. Confabulation is common during
the early phases of Korsakoff ’s syndrome but is unusual in the chronic phase of the condition.
Administration of thiamine during the acute Wernicke’s phase may prevent emergence of
Korsakoff ’s syndrome. Once the memory defect is established, however, thiamine has little effect
except to prevent further deterioration.

How well did you know this?
1
Not at all
2
3
4
5
Perfectly
24
Q

Which of the following is the least valuable clinical indicator of severity of
head injury?
A. Duration of retrograde amnesia
B. Glasgow Coma Scale
C. Duration of unconsciousness
D. Neurological lesions noted using an MRI
E. Duration of post-traumatic amnesia

A

A. There are several clinical indicators that predict severity of a head injury. They include
duration of retrograde amnesia, the depth of unconsciousness as assessed by the worst score
on the Glasgow Coma Scale (GCS), the duration of coma, neurological evidence of cerebral
injury, using an MRI or EEG, and the duration of post-traumatic amnesia. Of these, the least useful
clinical indicator is the duration of retrograde amnesia. Duration of post-traumatic amnesia is the
best marker of outcome. Patients with a post-traumatic amnesia of less than 1 week will have
minimal disability, while duration of more than 1 month is suggestive of enduring and signifi cant
disability. Other predictors of a bad outcome include previous head injury, older age, APOE e4
status, and alcohol dependence. Head injury can be classifi ed as mild wherein a GCS score
of 13 to 15 is likely to be associated with only a short duration of loss of consciousness (less
than 20 minutes) and a short post-traumatic amnesia (less than 24 hours). In moderate head
injury, GCS score 9 to 12 is likely to be associated with loss of consciousness of more than a
few minutes but less than 24 hours and a post-traumatic amnesia of more than 1 day but less
than 1 week. In severe head injury, a GCS score 3 to 8 is likely to be associated with a loss of
consciousness of more than 1 day or a post-traumatic amnesia of more than 1 week.

How well did you know this?
1
Not at all
2
3
4
5
Perfectly
25
Q

A 24-year-old patient is admitted to a head injury unit following a road
traffi c accident. He recovers well from acute neurological defi cits but
is diagnosed with post-concussion syndrome. Which of the following
statements pertaining to his condition is true?
A. There is a consistent relationship between severity of injury and the presence of postconcussion
syndrome.
B. Diplopia is an early symptom of post-concussion syndrome.
C. CT scans show brain lesions in up to 50% of patients in the fi rst week.
D. Psychological factors are more likely to play a role in illnesses of shorter duration.
E. There is no association between the presence of symptoms and compensatory claims

A

B. The term post-concussion syndrome (PCS) is used to describe a cluster of symptoms
that results in severe disability following mild head injury. There is no consistent relationship
between the prevalence of PCS and the severity of head injury. Sometimes a similar constellation
of symptoms may be seen in moderate and severe injury, where it is more likely to be attributed
to the actual brain damage. Symptoms are usually vague, but early symptoms may include
neurological complaints such as diplopia, dizziness, etc. Additional symptoms include cognitive
impairment, fatigue, anxiety, depression, and irritability. In general, most neurological symptoms
will have resolved by 2 to 6 months. Several observations support an organic basis, for example
diffuse microscopic axonal injury on post mortem, macroscopic brain lesions evident in 8–10%
of individuals on CT scan, subtle abnormalities on EEG, etc. Psychosocial factors play a part
in the syndrome, especially in those lasting longer than 1 year. This is greatest in those with
very mild head injuries and very chronic symptoms. There is an association between severity
of post-concussion symptoms and seeking compensation, but very few improve even after the
compensation.

How well did you know this?
1
Not at all
2
3
4
5
Perfectly
26
Q

A 30-year-old man who was involved in a road traffi c accident was
unconscious for 10 minutes. His CT scan was normal and he is now
conscious, but complaining of a bad headache. The family is concerned
about him developing seizures as his father has a history of epilepsy. What is
the next line of action?
A. Start phenytoin for 1 to 2 weeks
B. Start prophylactic carbamazepine for a year
C. Start long-term benzodiazepines
D. An abnormal EEG in this patient is an indication for starting prophylactic antiepileptic
medication
E. Antiepileptics are not indicated

A

E. Anticonvulsants are not indicated at this point in time, especially since the patient has
no symptoms suggestive of seizures. About 2 to 5% of all patients with mild, closed head injury
tend to develop long-term seizure disorder. This rises to about 10 to 20% in patients with severe,
closed head injury. A higher incidence of seizures has been seen in patients with depressed skull
fractures (15%), haematomas (30%), and penetrating brain wounds (50%). Early seizures, within
the fi rst week, are relatively benign and are only weak predictors of later epilepsy. This patient has
a mild, closed injury, and he is at a low risk for developing seizures, despite positive family history.
Randomized controlled studies have shown that the use of anticonvulsants does not prevent the
development of post-traumatic epilepsy beyond the fi rst week after injury. There is a limited role
for genetic predisposition in developing post-traumatic epilepsy. Those with the ApoE-ε4 allele
may be at higher risk for post-traumatic epilepsy

How well did you know this?
1
Not at all
2
3
4
5
Perfectly
27
Q

A 25-year-old patient presented with a history of recurrent, unilateral visual
disturbances that resolved completely, on-and-off episodes of pins and
needles in her left hand, and recent-onset bladder disturbances. Which of
the following statements regarding this illness is true?
A. The risk of her developing a major depressive disorder is 5–10% during her lifetime.
B. Her likelihood of developing suicidal ideation is similar to that in the general population.
C. She is 10 times more likely to develop a manic episode compared to the general
population.
D. Pathological laughing and crying is seen in around 10% of cases.
E. There is no risk of triggering a relapse of neurological condition with ECT.

A

D. This patient is most probably suffering from multiple sclerosis. The lifetime prevalence
of major depression in multiple sclerosis (MS) is around 50%. It is three to 10 times the rate in
the general population. Suicidal intent occurs in up to 30% of MS patients. This is linked to the
presence and severity of depression and degree of social isolation. Suicide rates in MS patients
are up to seven times higher than rates in the general population. Depression and suicide rates
are higher in MS than in most other neurologic disorders. In MS patients, the lifetime prevalence
of bipolar affective disorder is twice the prevalence in the general population. Pathological
laughing and crying is a syndrome that presents with inappropriate laughter without associated
happiness and inappropriate tears without associated sadness. Approximately 10% of MS patients
are affected, with varying degrees of severity. ECT is generally well tolerated by patients with MS,
but carries a risk of neurological relapse and exacerbation of the illness

How well did you know this?
1
Not at all
2
3
4
5
Perfectly
28
Q

Which of the following statements regarding cognitive impairment in
multiple sclerosis is true?
A. Cognitive defi cits are secondary to depressive symptoms.
B. Cognitive defi cits are closely related to physical disability and duration of illness.
C. Memory defi cits in multiple sclerosis are more apparent on recall compared to
recognition.
D. MMSE is a good test to screen for cognitive defi cits in multiple sclerosis.
E. Donepezil has not been found to be useful in improving memory in multiple sclerosis.

A

C. Around 50% of patients with MS have cognitive defi cits. Aphasia, apraxia, and agnosia,
which are characteristic of predominantly cortical diseases, are generally absent in MS, where
pathology is largely confi ned to subcortical white matter. Although patients with long-lasting
and advanced physical disability may also have severe cognitive impairment, the correlation
between cognitive dysfunction and disease characteristics (type and duration of MS) is usually
weak or modest. Cognitive defi cits are also independent of mood symptoms in MS. Defi cits in
working, semantic, and episodic memories have been reported. MS patients have diffi culty both in
acquiring and in retrieving information (although performance on recognition tests is better than
recall). Procedural memory is usually unaffected. Impaired attention and slowness of thinking is
another feature of MS. Frontal lobe defi cits may take the form of defi cits in conceptualization and
abstract thinking. At least one study has shown that donepezil is effective in improving cognitive
defi cits in MS.

How well did you know this?
1
Not at all
2
3
4
5
Perfectly
29
Q

Which of the following statements regarding pathological laughing and
crying is true?
A. It is always associated with motor defi cits such as pseudobulbar palsy.
B. Exaggerated crying and laughing is attributed to an underlying mood disorder.
C. Antidepressants have been found to be of no use in treatment.
D. It has been associated with frontal executive function defi cits.
E. By defi nition, patients cannot have a comorbid mood disorder.

A

D. Pathological laughter and crying is a symptom seen in MS, where approximately 10% of
MS patients are affected, with varying degrees of severity. This is similar to descriptions of the
pseudobulbar affect, although this symptom can be present without pseudobulbar palsy. Patients
are more likely to have frontally mediated cognitive defi cits. Commonly used scales to identify
and characterize this syndrome include the Pathological Laughter and Crying Scale and the
Centre for Neurologic Study–Lability Scale. The most common differential diagnosis is a mood
disorder, but patients with pathological crying exhibit the emotional display in the absence of a
pervasive and sustained elation or depressed mood. But when mood disorder and pathological
laughter and crying coexist, differentiation can be very diffi cult. TCAs and SSRIs have been found
to be effective in the treatment even if no depression is noted

How well did you know this?
1
Not at all
2
3
4
5
Perfectly
30
Q
Consciousness is preserved in which of the following types of seizures?
A. Tonic–clonic seizures
B. Simple partial seizures
C. Status epilepticus
D. Absence seizures
E. Complex partial seizures
A

B. Epilepsy is a common disorder, affecting approximately 1% of the population and
may involve individuals of any age. Seizures are convulsions that may be produced by a wide
variety of events, including alcohol and drug withdrawal syndromes, hypoglycaemia, transient
cerebral anoxia, and epileptic syndromes. Epilepsies are characterized by recurrent seizures and
their classifi cation is based on seizure type, age of onset, intellectual development, fi ndings on
neurological examination, and results of neuroimaging studies. Seizures are broadly classifi ed into
partial and general forms. Partial seizures are further divided into simple and complex. In simple
partial seizures consciousness is preserved; complex seizures are characterized by disturbances in
consciousness. Partial seizures pertain to one half or one particular area of the brain. Generalized
seizures involve both hemispheres from the beginning of the seizure but, at times, they may
be secondary to spread from a partial seizure. Tonic–clonic seizures and absence seizures are
examples of primary generalized seizures. They are usually associated with a loss of conscious
awareness.

How well did you know this?
1
Not at all
2
3
4
5
Perfectly
31
Q

A 45-year-old lady developed recurrent seizures with aura, automatism,
and lip smacking. Ictal EEG showed spike and sharp waves complex along
the right temporal region. Which of the following statements regarding her
diagnosis is not true?
A. The aura itself constitute a simple partial seizure.
B. This presentation is highly suggestive of complex partial seizure.
C. There is an increased chance of this lady developing mania in her lifetime.
D. A right-sided focus increases the risk of depression compared to mania.
E. There is a fi ve-times increase in the risk of suicide compared to the general population.

A

D. Left-sided foci are associated with an increased risk of depression and right-sided foci
with an increased risk of mania. Mood disorders are the most common type of psychopathology
encountered in patients with epilepsy. Prevalence rates of depression range from 30 to 50% in
patients with epilepsy. With intractable disorders, up to 60% have lifetime histories of depressive
syndromes. In contrast to the incidence of depression in epilepsy, the incidence of mania and
bipolar disorder are at normal or near-normal levels. Some retrospective chart reviews state a
lifetime prevalence of 20%. The incidence of suicide in epilepsy patients is fi ve to 10-fold greater
than in the general population. In those with temporal lobe epilepsy, suicide rates are around
25 times that of the general population

How well did you know this?
1
Not at all
2
3
4
5
Perfectly
32
Q
Features suggestive of Geschwind’s syndrome include all except
A. Circumstantiality
B. Hypographia
C. Hyper-religiosity
D. Viscosity
E. Increased aggression
A

B. Geschwind’s syndrome is an eponymous syndrome of interictal behaviour/personality
disorder which has been described in temporal lobe epilepsy (TLE). Clinical features of this
syndrome include preoccupation with philosophical and religious concerns, anger, excessive
emotionality, viscosity (noted especially in speech), circumstantiality, altered sexuality, and
hypergraphia. Recent reviews state that personality traits, rather than a personality disorder per
se, seems more likely in these disorders and they tend to resemble the cluster C category of
disorders in DSM-IV

How well did you know this?
1
Not at all
2
3
4
5
Perfectly
33
Q

A 23-year-old patient previously diagnosed with epilepsy presents to
casualty with intractable seizures following the breakup of a relationship.
She has been compliant on her medications. The neurologist suspects
psychogenic non-epileptic seizures. Which of the following statements is
true with regard to her condition?
A. A postictal prolactin elevation of two times the baseline level is reliable in diagnosing
true seizures.
B. Up to 80% of patients with seizures have psychogenic non-epileptic seizures.
C. Less than 5% of cases with intractable seizures have psychogenic non-epileptic seizures.
D. Presence of tongue bite and incontinence is diagnostic of true seizures.
E. Video EEG recording is the gold standard for diagnosing psychogenic non-epileptic
seizures.

A

E. Psychogenic non-epileptic seizures were previously referred to as ‘pseudoseizures’. They
are seizure-like behavioural events that occur in the absence of abnormal electrical discharge
in the brain. The gold standard for diagnosis is video-EEG monitoring. People who present with
non-epileptic seizures commonly have comorbid epilepsy. Nearly 30 to 50% of patients who
have non-epileptic seizures have epilepsy and 20 to 60% of patients who have epilepsy have
non-epileptic seizures. The average age of onset is between 20 and 30 years and it is three times
more common in women than men. Prevalence rates of a history of sexual abuse in non-epileptic
seizures range from 25 to 75%. An elevated prolactin level (usually two times baseline or three
standard deviations above normal) could be due to seizures or any neurological event, such as
syncope. Some patients with pseudoseizures may have modest elevations in prolactin levels.
A normal prolactin may not always be diagnostic of pseudoseizures, since it is frequently normal
in partial seizures and sampling may be mistimed following convulsions

How well did you know this?
1
Not at all
2
3
4
5
Perfectly
34
Q

Early onset of major depression is most commonly associated with stroke
pertaining to which of the following regions of the brain?
A. Right anterior
B. Right posterior
C. Left anterior
D. Left posterior
E. Bilateral posterior

A

C. Historically, left anterior stroke has been associated with depression. This has been
questioned in more recent times—including a few meta-analyses that did not show such a
relationship. The laterality hypothesis of poststroke depression may hold true only in the acute
stage of illness of less than 2 months’ duration. As time passes, the chance of getting a depressive
episode is equal in all kinds of stroke. Major depression occurs in approximately 10–25% of
patients. Anxiety occurs without depression in up to 10%. Apathy occurs in 20% of patients.
Anosognosia with denial of illness is present in 25–45% of patients, particularly those with right
posterior lesions. Catastrophic reactions appear in approximately 20% and emotional lability is
present in 20%. The mean duration of major depression appears to be about 9 months, but can
be chronic, lasting for years in hospitalized patients.

How well did you know this?
1
Not at all
2
3
4
5
Perfectly
35
Q

A 67-year-old business man is admitted to a stroke unit. He is having
signifi cant aphasia. He has episodes of anger outburst when someone tries
to communicate with him. Which of the following is false regarding this
‘catastrophic reaction’?
A. Family history of psychiatric disorders is more common in those with catastrophic
reaction.
B. It is associated with the presence of major depression.
C. The reaction is mostly secondary to the presence of aphasia.
D. Patients are more likely to have a personal history of psychiatric disorders.
E. Higher frequency of basal ganglia lesions may be seen.

A

C. Goldstein instigated the term ‘catastrophic reaction’ to describe a cluster of symptoms
characterized by aggressive outbursts in patients with brain injury. It was ascribed to the inability
of the person to cope with the physical/cognitive defi cit. An important study with respect to
catastrophic reaction was conducted by Starkstein et al. in 1993. The major fi ndings of this study
are as follows. Catastrophic reaction occurs in around 20% of stroke patients. It is associated
with a personal and family history of psychiatric illness. It is also signifi cantly associated with the
presence of poststroke depression. It is more common in anterior subcortical lesions and lesions
involving the basal ganglia. The reaction is not merely a frustration reaction to the presence of
aphasia or cognitive defi cits; it could be present as a symptom on its own or as a behavioural
symptom in a subgroup of depressed patients with anterior subcortical damage.

How well did you know this?
1
Not at all
2
3
4
5
Perfectly
36
Q

Which of the following statements about poststroke depression is true?
A. Younger age predisposes to poststroke depression.
B. Cortical atrophy prior to stroke predisposes to poststroke depression.
C. Male sex is a risk factor for poststroke depression.
D. Lower educational status is a risk factor for poststroke depression.
E. Lower socioeconomic status is a risk factor for poststroke depression

A

B. On the basis of the fact that not all patients with a left anterior or a right posterior lesion
develop depression, other premorbid factors were studied by Starkstein et al. Along with the
presence of family history of affective disorders in those who developed poststroke depression,
they also found that there was no signifi cant relationship between the presence of depression
and demographic variables such as age, sex, education, socioeconomic status, etc. Presence
of premorbid cortical atrophy was found to be a risk factor for depression. Similarly, cortical
atrophy also predicts mania.

How well did you know this?
1
Not at all
2
3
4
5
Perfectly
37
Q

A 67-year-old patient with stroke has left inferior quadrantanopia, left
hemineglect, and dressing apraxia with mild hemiparesis on neurological
examination. Which artery is most likely to be involved in the stroke?
A. Posterior cerebral artery
B. Middle cerebral artery
C. Anterior cerebral artery
D. Common carotid artery
E. Internal carotid artery

A

B. The middle cerebral artery supplies most of the cortical grey matter, including the
parietal cortex. The inferior parietal lobe includes the upper part of the optic radiation which
carries fi bres from upper half of the retina and hence lesions of this area produce inferior
quadrantanopia. Parietal lobe damage also explains the hemineglect, mild hemiparesis, and
dressing apraxia. Carotid artery syndrome usually presents with amaurosis fugax, the feature
distinguishing it from the middle cerebral artery syndrome. Amaurosis fugax is transient, painless
monocular blindness, usually due to emboli either from the large arteries or the heart itself.
Occlusions of the coronaries usually occur at the bifurcation of the common carotid.

How well did you know this?
1
Not at all
2
3
4
5
Perfectly
38
Q
Which of the following stages of sleep is characterized by more than 50%
delta activity in the EEG?
A. Stage 1 NREM
B. Stage 2 NREM
C. Stage 3 NREM
D. Stage 4 NREM
E. REM sleep
A

D. Stage 4 NREM sleep is characterized by more than 50% delta activity. When the delta
activity ranges from 20 to 50%, the person is in stage 3 NREM sleep. K complexes and sleep
spindles along with delta waves of less than 20% is noted in stage 2. Stage 2 is also the longest
sleep stage through the night, comprising almost 50% of adult sleep. Stage 1 is characterized by
gradual slowing of the alpha wave (less than 50% alpha activity). This is the sleep onset. Stage W
(wakefulness) is characterized by predominantly alpha waves posteriorly with low voltage mixed
frequency beta waves anteriorly. REM sleep constitutes around 20–25%. Normally, much less time
is spent in stage W and stage 1.

How well did you know this?
1
Not at all
2
3
4
5
Perfectly
39
Q
All the following are features of REM sleep except
A. Low brain oxygen consumption
B. High cerebral blood fl ow
C. Penile erection
D. Absent electrodermal activity
E. Dream-like mental state
A

A. Characteristics of REM sleep include variable heart and breathing rate, high oxygen
consumption and cerebral blood fl ow, penile erections (morning erections due to high levels
of REMs), increased vaginal blood fl ow and uterine activity, absent electrodermal activity,
poikilothermic state, and dream-like mental activity. In contrast, NREM sleep is characterized
by regular, slow heart and breathing rate, low cerebral blood fl ow and O2 consumption, absent
penile blood fl ow, and thought-like mental activity. Muscular tone is maintained in NREM sleep
and atonia is seen in REM.

How well did you know this?
1
Not at all
2
3
4
5
Perfectly
40
Q

Regarding sleep terror, which of the following statements is false?
A. It is associated with REM sleep disturbance.
B. Vocalizations may occur during the episode.
C. There is usually amnesia for these episodes.
D. It becomes exacerbated by sleep deprivation.
E. It is associated with psychopathology in adults.

A

A. Characteristics of REM sleep include variable heart and breathing rate, high oxygen
consumption and cerebral blood fl ow, penile erections (morning erections due to high levels
of REMs), increased vaginal blood fl ow and uterine activity, absent electrodermal activity,
poikilothermic state, and dream-like mental activity. In contrast, NREM sleep is characterized
by regular, slow heart and breathing rate, low cerebral blood fl ow and O2 consumption, absent
penile blood fl ow, and thought-like mental activity. Muscular tone is maintained in NREM sleep
and atonia is seen in REM.

Sleep terrors occur in slow-wave sleep (stage 3 and 4) unlike nightmares which occur
in REM sleep. Sleep terrors are characterized by a sudden arousal with intense fearfulness, often
associated with a sharp scream. The subject may sit up in bed, may vocalize unintelligibly, and
waking the individual leads to confusion. There is amnesia for the episode and unlike nightmares
it is very rarely associated with vividly recalled dreams and images. A familial pattern has been
reported. In children, night terrors may be transient but in adults they may be associated with
other psychopathology.

How well did you know this?
1
Not at all
2
3
4
5
Perfectly
41
Q

A 25-year-old man complains of excessive daytime sleepiness. He loses
balance and falls down every time he laughs at a joke. He also complains
of seeing ‘ghosts’ while falling asleep. Which of the following is likely to be
found in this patient?
A. A sleep-onset slow wave stage
B. Excess of hypocretin in the hypothalamus
C. Seizure activity on electroencephalography
D. Episodes of sleep paralysis
E. Absence of REM on polysomnography

A

D. This patient is likely to have narcolepsy. Narcolepsy is a disorder of unknown aetiology.
It consists of the tetrad of excessive day time sleepiness, cataplexy, sleep paralysis, and hypnagogic
hallucinations. Polysomnography typically shows sleep-onset REM stage. Cataplexy refers to
sudden loss of muscular tone, often seen in association with emotional reactions in those with
narcolepsy. An abnormality in the hypocretin neurones in the lateral hypothalamus has been noted
in those with narcolepsy. Hypocretin (orexin) is a highly excitatory peptide hormone secreted
from the hypothalamus. This is necessary to maintain wakefulness and it also increases appetite.
Narcolepsy, especially cataplexy, is considered to be a hypocretin defi ciency syndrome. SSRIs and
TCAs remain the treatment of choice currently. Modafi nil is also being tried as a treatment

How well did you know this?
1
Not at all
2
3
4
5
Perfectly
42
Q

All of the following increases the risk of developing dementia in those with
Parkinson’s disease except
A. Older age group
B. Greater severity of motor disturbances
C. Longer duration of Parkinson’s disease
D. Being female
E. Signifi cant functional disability

A

D. Dementia is estimated to occur in 27% of patients with Parkinson’s disease (PD).
Dementia has been associated with older age, greater PD severity, hallucinations, longer duration
of PD, greater disability, and male gender. Causes of dementia in PD include Lewy body pathology,
dopamine depletion, coexisting AD, and other conditions. Reduced fl uorodopa uptake in the
frontal cortex and caudate nucleus, and in mesolimbic pathways are predictors of cognitive
impairment. Temporoparietal cortical hypometabolism also predicts cognitive impairment.
Donepezil has been found to be useful in two separate double-blind trials in patients with PD.

How well did you know this?
1
Not at all
2
3
4
5
Perfectly
43
Q
Which of the following is considered as a ‘Parkinson plus’ syndrome?
A. Wilson’s disease
B. Fredreich’s ataxia
C. Progressive supranuclear palsy
D. Amyotrophic lateral sclerosis
E. Guillain–Barré syndrome
A

C. Progressive supranuclear palsy is considered as a Parkinson plus syndrome. It is
distinguished from Parkinson’s disease by the presence of early broad-based and stiff gait
disorder (axial greater than limb rigidity in extension) with backward falls, and supranuclear
gaze palsy with slow vertical saccades and diffi culty looking down (and hence the falls). Falls
are very common in these patients and are an important cause of morbidity. Patients are
prone to develop various psychiatric complications, including cognitive dysfunction and mood
disorders. Cholinesterase inhibitors have not been particularly useful in treating patients with
progressive supranuclear palsy associated dementia. Other Parkinson plus syndromes include
multisystem atrophy (called Shy–Drager syndrome when associated with autonomic failure),
olivopontocerebellar atrophy, and corticobasal degeneration.

How well did you know this?
1
Not at all
2
3
4
5
Perfectly
44
Q

A 64-year-old man presents with sudden-onset blindness that started as a
‘curtain coming down’ and he lost his vision completely for a few minutes.
Within 15 minutes this improved and was restored to full, normal vision.
The origin of emboli in this case is most likely to be at
A. Posterior cerebral artery
B. Anterior cerebral artery
C. Internal carotid artery
D. Anterior communicating artery
E. Middle cerebral artery

A

C. In this case the origin of emboli must be at the internal carotid artery. The ophthalmic
artery, a branch of the internal carotid artery, is blocked transiently, producing the symptoms
described in the question. This is called as amaurosis fugax which translates to ‘fl eeting darkness’.
It is related to transient but sudden monocular visual loss as a result of decreased retinal
circulation. It is a type of transient ischaemic attack and could be a harbinger of a cerebrovascular
accident. Compromise of posterior circulation usually leads to cortical blindness, often with
macular sparing.

How well did you know this?
1
Not at all
2
3
4
5
Perfectly
45
Q

A 21-year-old lady is found wandering at a public place. She is unaware of
her address or any other personal details. She was admitted and later found
to be on the missing persons register at a police station 100 miles away.
After 4 weeks, she regains normal memory and remembers having lost her
mother in a fi re accident 6 weeks ago. Which of the following is true about
the nature of her memory problems?
A. Total amnesia for past events may be seen during the episode.
B. No amnesia for the episode will be present following recovery.
C. A vascular aetiology is most likely.
D. Inability to learn new materials will be seen during the episode.
E. Episodes are often accompanied by other neurological symptoms.

A

A. The given history is consistent with dissociative fugue. Fugue states are associated with
stressful life events wherein total amnesia for the recent past can be seen. More often the
amnesia related to dissociation is circumscribed to events of personal importance. During the
fugue, the patient may retain normal functional activities and may even learn new verbal and nonverbal
materials. Following recovery from the fugue, one may not remember the activities carried
out during the fugue state.

How well did you know this?
1
Not at all
2
3
4
5
Perfectly
46
Q

A 63-year-old man with alcohol dependence suffers a serious head injury.
On recovery he is found to have unusual behaviours. When a tooth brush
is placed in front of him, he immediately begins to brush his teeth, even in
entirely inappropriate contexts. He is exhibiting
A. Alien hand syndrome
B. Klüver–Bucy syndrome
C. Utilization behaviour
D. Executive dysfunction
E. Balint’s syndrome

A

C. This is an example of utilization behaviour. A patient with utilization behaviour will be
forced to ‘utilize’ objects presented to him despite the absence of obvious need for such usage.
An extreme form of this is seen in ‘environmental dependence syndrome’—the patient becomes
compelled to make use of all that is seen in his immediate environment resulting in an array of
serial complex behaviour. Klüver–Bucy syndrome occurs in the context of bilateral temporal
lobe damage. It is associated with hyperorality, inappropriate sexuality, and increased exploratory
behaviour (hypermetamorphosis). Alien hand syndrome refers to loss of control of limb
movements resulting in ‘automatic’ coordinated hand movements. The patient is usually aware of
this and may try to exercise control using the other hand. Balint’s syndrome is characterized by
oculomotor apraxia, optic ataxia, and simultanagnosia

How well did you know this?
1
Not at all
2
3
4
5
Perfectly
47
Q

The most probable site of a lesion for the patient described in
Question 47 is
A. Frontal lobes
B. Dominant parietal lobe
C. Occipitoparietal junctions bilaterally
D. Bilateral amygdala
E. Corpus callosum

A

A. Utilization behaviour is seen in patients with frontal lobe damage. Dominant parietal
lobe lesions result in apraxia, right–left confusion, acalculia, and fi nger agnosia. Balint’s syndrome
occurs in bilateral parieto-occipital damage.

How well did you know this?
1
Not at all
2
3
4
5
Perfectly
48
Q

A 32-year-old woman with complex partial seizures is referred to a
psychiatrist to exclude psychosis. She experiences olfactory hallucinations
and intense anxiety. Which of the following is not correct with regard to
complex partial seizures?
A. Temporal lobe is the most common site of origin.
B. Ictal hallucinations are often accompanied by emotional reactions.
C. Patients are often aware of the unreal nature of the hallucinations.
D. Irritability is the most common emotional reaction accompanying the aura.
E. Déjà vu is a well-known phenomenon occurring in complex partial seizures.

A

D. Temporal lobe is the most common site of origin of complex partial seizures; in very
few cases parietal focus has been demonstrated. When accompanied by hallucinations, intense
emotional reactions may be seen during the seizure. Fear is the most common emotion noted.
Curiously, patients are often aware of the unreal nature of their hallucinatory experiences, but
this awareness is not specifi c to complex seizures. Pathology of familiarity characterized by déjà
vu and jamais vu are commonly reported in temporal lobe epilepsy.

How well did you know this?
1
Not at all
2
3
4
5
Perfectly
49
Q

A 78-year-old woman presents with fl uent progressive aphasia with
preservation of new learning and orientation. On follow up she is observed
to have progressive diffi culties in understanding the meaning of words used
during normal conversation. She is most likely to have
A. Alzheimer’s dementia
B. Semantic dementia
C. Lewy body dementia
D. Broca’s aphasia
E. Vitamin B12 defi ciency

A

B. Progressive fl uent aphasia early in the course of a dementing illness is a feature of
semantic dementia. Semantic dementia is a type of frontotemporal degenerative disorder. The
pathological fi nding is predominantly frontotemporal degeneration with ubiquitin inclusions.
Motor neurone disease type inclusions may also be noted. Semantic memory refers to
representation of meanings, understanding concepts, and knowledge unrelated to temporal
events (cf. episodic memory). Focal cortical defi cits, especially progressive aphasia, can be
presenting features of Alzheimer’s disease but this is uncommon.

How well did you know this?
1
Not at all
2
3
4
5
Perfectly
50
Q
Which of the following can be used to test premorbid IQ in patients with
neurological damage?
A. National Adult Reading Test
B. Rivermead Behavioural Memory Test
C. Weschler’s Memory Scale
D. Mini Mental State Examination
E. Minnesota Multiphasic Inventory
A

A. The National Adult Reading Test (NART) has been widely used as a measure of
premorbid IQ. The usefulness of NART as a measure of premorbid IQ is based on two
assumptions:
1. Reading ability is relatively independent of brain damage.
2. Ability to read irregular words from a list is a strong predictor of intelligence in the normal
population.
Hence in those with brain damage, irrespective of the diagnosis, NART can be used to estimate
the most probable IQ level before becoming ill. However, the notion that the NART score is
relatively independent of brain damage has come under scrutiny of late. Studies in Alzheimer’s
dementia and Korsakoff ’s syndrome have indicated deterioration in reading ability, leading to an
underestimated premorbid IQ.

How well did you know this?
1
Not at all
2
3
4
5
Perfectly
51
Q

A 55-year-old man with history of long-standing, untreated hypertension
is brought to A&E by his wife following 3 hours of ‘confusion’. He was
repeatedly questioning her and was not able to remember what he was
doing 30 minutes ago. He is also unaware of events of the past 2 weeks,
despite remembering them until 3 hours ago. Neurological examination
is otherwise unremarkable and he has no psychiatric history. The episode
resolves by itself within 24 hours. Which of the following is false with regard
to his condition?
A. Immediate memory will be intact.
B. Anterograde amnesia will be predominant.
C. Patchy and inconsistent retrograde amnesia will be seen.
D. Visuospatial and problem-solving functions will be affected.
E. Rapid recovery occurs in most individuals.

A

D. This history is consistent with transient global amnesia (TGA). Sudden-onset amnesia
with inability to form new memories of current events and a variable degree of retrograde
amnesia is seen. The entire episode lasts for hours to days and on recovery the extent of
retrograde amnesia shrinks and almost intact memory for events that happened before the
episode is restored, but a dense amnesia persists for the events during the episode (24 hours in
this case) even after full recovery. During the episode itself, the procedural memory, visuospatial
functions, and problem-solving ability are intact; the patients may even be able to drive during the
episode.

How well did you know this?
1
Not at all
2
3
4
5
Perfectly
52
Q

Abnormalities in which of the following vascular territories is implicated in
the presentation described in Question 52?
A. Anterior cerebral circulation
B. Middle cerebral circulation
C. Posterior cerebral circulation
D. Cortical venous sinus outfl ow
E. Middle meningeal circulation

A

C. TGA is thought to be vascular in origin. Posterior cerebral circulation (vertebrobasilar
insuffi ciency) is implicated, which supplies signifi cant part of hippocampal and other medial
temporal regions. Migrainous or epileptic aetiology has not been entirely disproved. Obstruction
to cortical sinuses may be related to idiopathic intracranial hypertension in some cases. The
middle meningeal artery may be injured in skull fractures, often becoming a source of extradural
haematoma.

How well did you know this?
1
Not at all
2
3
4
5
Perfectly
53
Q

A patient with long-standing, uncontrolled type 2 diabetes presents with
anterior spinal artery occlusion. Which of the following sensations carried
by the spinal cord is most likely to be affected?
A. Proprioception
B. Vibration
C. Pain
D. Joint position
E. Light touch

A

C. The anterior spinothalamic tract is supplied by the anterior spinal artery. The
spinothalamic tract carries pain and temperature sensations. Posterior tracts such as dorsal
columns of gracilis and cuneatus carry joint sense, light touch, proprioception, and vibration
sensations. Infarction of the spinal cord usually involves the territory of the anterior spinal
artery—the ventral two-thirds of the spinal cord.

How well did you know this?
1
Not at all
2
3
4
5
Perfectly
54
Q
Which of the following structures is a part of cerebellum?
A. Dentate nucleus
B. Red nucleus
C. Substantia nigra
D. Subthalamic nucleus
E. Insular cortex
A

A. The cerebellum has an external cortical grey matter and the deep cerebellar nuclei.
There are four deep nuclei: dentate, globose, emboli-form, and fastigial nuclei. Mossy fi bres and
climbing fi bres provide the major input into the cerebellum. Substantia nigra and subthalamic
nucleus are part of basal ganglia. Red nucleus is an upper brainstem nucleus seen at the level of
tegmentum. Similar to substantia nigra, red nucleus also contains iron pigments. It is involved in
motor coordination

How well did you know this?
1
Not at all
2
3
4
5
Perfectly
55
Q

Which of the following is true with respect to pseudobulbar palsy?
A. It is caused by diffuse brain stem damage.
B. It is often accompanied by fl accid tongue.
C. Jaw jerk is exaggerated.
D. It is seen in poliomyelitis.
E. Frontal release signs are inconsistent with the diagnosis

A

C. Pseudobulbar palsy is also known as spastic bulbar palsy. It usually results from
bilateral frontal damage. This may be due to vascular, demyelinating, or motor neurone disease
(amyotrophic lateral sclerosis). Diffuse brainstem damage (bulb) will produce lesions in the
cranial nerve nuclei, causing bulbar palsy. Pseudobulbar palsy is an upper motor neurone type of
lesion. Bulbar palsy produces lower motor neurone damage. Hence increased tone (producing
spastic tongue), exaggerated tendon refl exes (brisk jaw jerk), and lack of fasciculations are
notable. Diffuse frontal damage may produce frontal release signs. Poliomyelitis, diptheria, and
Guillain–Barré syndrome are known causes. The most common cause of progressive bulbar palsy
is motor neurone disease

How well did you know this?
1
Not at all
2
3
4
5
Perfectly
56
Q
Lesions of the subthalamic nucleus are associated with
A. Chorea
B. Hemiballismus
C. Tics
D. Epilepsy
E. Visual neglect
A

B. Lesions of the subthalamic nucleus are associated with hemiballismus. Lesions of the
caudate nucleus are associated with chorea. Disturbances in the GABA system of caudate
nucleus are noted in Huntington’s disease. Similarly, damage to the caudate nucleus is implicated
in Sydenham’s chorea seen in streptococcal infection. Parkinsonian movement disturbances,
especially bradykinesia, are associated with damage to the substantia nigra

How well did you know this?
1
Not at all
2
3
4
5
Perfectly
57
Q
Which of the following neuropsychological tests is primarily used to detect
errors in set-shifting capacity?
A. Tower of London
B. Rey Osterrieth Complex Figure Test
C. Wisconsin Card Sorting Test
D. Word Fluency Test
E. Letter Cancellation Test
A

C. Set shifting is an executive function. In the Wisconsin Card Sorting Test, abstract
reasoning and fl exibility in problem solving are tested. In this test, cards of different colour, form,
and number are available. Patients are asked to sort the cards into groups according to varying
categories (colour only, form only, or number only) as requested by the examiner. This measures
the capacity for abstract thinking and set-shifting ability (cognitive fl exibility). Tower of London
is a problem-solving test; it involves frontal and basal ganglia function but does not directly test
set-shifting ability. Rey Osterrieth fi gure is a test of visual memory wherein a complex geometric
fi gure is given to be copied, followed by immediate reproduction from memory and reproduction
after a delay.

How well did you know this?
1
Not at all
2
3
4
5
Perfectly
58
Q

A 72-year-old man is afflicted with stroke. He is not able to identify objects
with their correct names but is able to demonstrate the usage correctly.
When the correct name is given to him, he is able to recognize it correctly.
He is suffering from
A. Motor aphasia
B. Apraxia
C. Anomic aphasia
D. Sensory aphasia
E. Abulia

A

C. Anomic (or nominal) aphasia presents with inability to name objects and body parts.
Patients have fl uent speech, intact repetition, intact comprehension, reading, and writing. Nominal
aphasia often presents together with, or may follow, recovery from other forms of aphasia.
Nominal aphasia is not very useful for lesion localization. It is also noted in early Alzheimer’s
disease.

How well did you know this?
1
Not at all
2
3
4
5
Perfectly
59
Q
The blood supply to the hippocampus comes from the
A. Basilar artery
B. Anterior communicating artery
C. Anterior cerebral artery
D. Anterior choroidal artery
E. Lenticulostriate arteries
A

D. The blood supply to the rostral third of the hippocampus comes from the anterior
choroidal artery, which is a direct branch of the internal carotid artery. It does not take part in
the circle of Willis anastamosis. The occipital two-thirds are supplied by hippocampal branches,
the posteromedial choroidal artery, and the inferior temporal branches of the posterior cerebral
artery.

How well did you know this?
1
Not at all
2
3
4
5
Perfectly
60
Q

In early Alzheimer’s disease, widespread loss of nerve cells is most
pronounced in which of the following structures?
A. Layer III of cerebral cortex
B. Layer IV of entorhinal cortex
C. Layer I of cerebral cortex
D. Layer II of entorhinal cortex
E. Layer IV of cerebral cortex

A

D. Early in the Alzheimer’s disease neuronal loss is most pronounced in layer II of the
entorhinal cortex of the hippocampus. The parahippocampal gyri and subiculum are also affected.
This extends to anterior nuclei of the thalamus, septal nuclei, amygdala, and monoaminergic
systems of the brainstem are also depleted. The cholinergic neurones of the nucleus basalis of
Meynert are also reduced. In cerebral cortex, most pronounced loss occurs with respect to
pyramidal neurones and astrocytic proliferation follows as a compensatory or reparative process,
most prominently in layers III and V.

How well did you know this?
1
Not at all
2
3
4
5
Perfectly
61
Q
Glutamate-induced excitotoxicity is proposed as a cause of which of the
following conditions?
A. Huntington’s disease
B. Crutzfeld–Jakob disease
C. Wilson’s disease
D. Korsakoff ’s syndrome
E. Weber’s syndrome
A

A. Excessive stimulation of glutamate receptors leads to an increase in intraneuronal
calcium and nitric oxide. Calcium activates proteases that could destroy the neurone from
within. Memantine is an NMDA antagonist used in the treatment of Alzheimer’s disease, based
on the excitotoxicity hypothesis. This mechanism may be applicable for Parkinson’s disease too.
In Huntington’s disease, an expansion of the polyglutamine region of huntingtin takes place
due to the disease-causing mutation. Hence the mutant huntingtin protein accumulates in the
nuclei of neurones, preferentially in striatum and cortex. These aggregates may be directly
toxic to some extent, but predominant striatal loss, as opposed to cortical loss, may be due to
glutamate-mediated excitotoxicity. Huntingtin accumulation may render cells unusually sensitive
to glutamate-mediated damage

How well did you know this?
1
Not at all
2
3
4
5
Perfectly
62
Q

Which of the following is a feature of occlusion of the right-sided posterior
inferior cerebellar artery?
A. Left-sided loss of facial pain sensation
B. Left-sided loss of facial temperature sensation
C. Loss of pain sensation on the right side of the body
D. Loss of temperature sensation on the left side of the body
E. Mydriasis of the right eye

A

D. Posterior inferior cerebellar artery occlusion leads to Wallenberg’s syndrome. The
resulting signs and symptoms are attributed to infarction of a wedge of lateral medulla that
contains vestibular nuclei, descending sympathetic tract, spoinothalamic system (carrying pain
and temperature from contralateral side of body), descending fi fth nerve tract and nucleus, and
ninth and tenth nerve fi bres of same side. This leads to ipsilateral Horner’s syndrome (miopsis,
anhidrosis, and ptosis due to sympathetic damage), ipsilateral loss of face sensation (fi fth nerve
damage), dysphagia, hoarseness, loss of gag refl ex (ninth/tenth nerve damage), and contralateral
loss of pain and temperature over half of the body.

How well did you know this?
1
Not at all
2
3
4
5
Perfectly
63
Q

A 65-year-old man presents with memory diffi culties and loss of balance.
He has signifi cant, new-onset urinary incontinence. CT scan of the brain
shows dilated ventricles but no signifi cant widening of sulci. The most likely
diagnosis is
A. Normal-pressure hydrocephalus
B. Alzheimer’s dementia
C. Lewy body dementia
D. Benign intracranial hypertension
E. Alcoholic dementia

A

A. The age of the patient, the triad of memory diffi culties, loss of balance, and urinary
incontinence, and the neuroimaging fi ndings suggest normal-pressure hydrocephalus (NPH).
NPH is not a hydrocephalus in the true sense—there is no increase in intracranial pressure when
lumbar puncture is carried out. Following certain meningeal insults, secondary to subarachnoid
haemorrhage, head trauma, or resolved meningitis, an increase in intracranial pressure may
develop but reach a stable stage where formation of CSF diminishes and equilibrates with
absorption, which increases proportionate to the pressure. Once this equilibrium is reached
there must be a gradual fall in pressure, although at a high normal level. In some patients, this high
normal intracranial pressure of 150 to 200 mm H2O leads to manifestation of NPH.

How well did you know this?
1
Not at all
2
3
4
5
Perfectly
64
Q

During polysomnographic recording of a patient with sleep disturbances,
it is observed that his heart rate and blood pressure are lower than that
recorded during normal wakefulness. His muscle tone is also notably low.
Which of the following is true with respect to his physiological state?
A. Vivid memory of dreams occur at this stage.
B. If awakened from this stage there will be some degree of confusion.
C. Penile erection occurs automatically at this stage of sleep.
D. High cerebral blood fl ow is seen at this stage.
E. In adults, 25% of sleeping time is spent in this stage of sleep.

A

B. The presence of low heart rate, muscle tone, and blood pressure is suggestive of NREM
sleep. At this stage of sleep if a person is awakened, he will be confused. He may not recollect
the instance of awakening in the morning. A normal adult spends nearly 75% of sleep in various
NREM stages, while the remaining 25% is REM sleep. Penile erection, high cerebral blood fl ow,
and vividly recalled dreams are features of REM stage

How well did you know this?
1
Not at all
2
3
4
5
Perfectly
65
Q
The frequency of alpha waves seen in EEG recordings is
A. >13 Hz
B. 8–12 Hz
C. 4–8 Hz
D. 0.5–4 Hz
E. <0.5 Hz
A

B. Alpha, beta, delta, and theta are four important wavelets in EEG when awake. Alpha waves
are predominant, especially posteriorly when the eyes are closed; they occur at a frequency of
8 to 13 Hz. Beta waves are sometimes seen in normal EEG over central, anterior regions; they
occur at a frequency higher than 13 Hz. Theta activity is seen infrequently when awake but often
when a subject is drowsy or sleeping. Excessive theta when awake is abnormal. Delta waves
(frequency less than 3.5 Hz) are normally seen only in deep sleep and are pathological if noted in
adult waking EEG. With ageing, slow waves become more common in EEG.

How well did you know this?
1
Not at all
2
3
4
5
Perfectly
66
Q
Diffuse fl attening of EEG with low-amplitude theta and delta waves
is seen in
A. Huntington’s disease
B. Alzheimer’s dementia
C. Hepatic encephalopathy
D. Delirium tremens
E. Crutzfeld–Jakob disease
A

A. Huntington’s disease is characterized by diffuse fl attening or loss of alpha waves in EEG.
In a study conducted in a group of 95 patients with Huntington’s chorea, 31 showed little activity
of any kind (fl at trace EEG) and in particular no alpha rhythm above 10 μV in amplitude was seen.
There was a statistically signifi cant association between cortical atrophy, especially the frontal
lobe, and a ‘low voltage’ EEG in the same study. Such low-voltage records, though not specifi c for
Huntington’s chorea, are rare in other neurological disorders

How well did you know this?
1
Not at all
2
3
4
5
Perfectly
67
Q
Which of the following functions is mediated by endogenous cannabinoids?
A. REM sleep induction
B. Motor coordination
C. Peripheral sympathetic modulation
D. Gut motility
E. Mediation of intraocular pressure
A

E. Two types of cannabinoid receptors, central (CB1) and peripheral (CB2), have been
identifi ed. Both receptors bind to exogenously administered tetrahydrocannabinol (THC),
present in marijuana. Anandamide (from the Sanskrit word ‘ananda’ for bliss) is chemically
N-arachidonoylethanolamine (arachidonic acid and ethanolamine derivative). It is a weak
endogenous cannabinoid ligand. 2-arachnidonylglycerol is a strong endogenous ligand for the
cannabinoid receptor. Endogenous cannabinoids exhibit intraocular pressure-lowering effects.
They also decrease motor activity level and relieve pain. Anandamides are demonstrated in the
thalamus, with a putative role in pain-related neurotransmission.

How well did you know this?
1
Not at all
2
3
4
5
Perfectly
68
Q

A 22-year-old man is diagnosed with craniopharyngioma. He is experiencing
symptoms due to the tumour pressing upon adjacent brain tissue. Which of
the following visual defect is characteristic of this tumour?
A. Tunnel vision
B. Homonymous hemianopia
C. Binasal hemianopia
D. Bitemporal hemianopia
E. Superior quandrantonopia

A

D. Bitemporal hemianopia is secondary to chiasmatic lesions. Pituitary tumours
characteristically cause bitemporal hemianopia. Craniopharyngioma is a benign epithelioid
tumour arising from remnants of Rathke’s pouch at the junction of the infundibular stem and
pituitary. It lies above the sella turcica and so exerts pressure effects on the optic chiasm
leading to bitemporal hemianopia. Tunnel vision is a result of extensive peripheral fi eld defects.
Quadrantanopias usually result from damage to the optic radiation beyond the chiasma. Parietal
lesions result in inferior, while temporal lesions result in superior, quadrantanopia.

How well did you know this?
1
Not at all
2
3
4
5
Perfectly
69
Q
Priapism is a side-effect associated with which of the following?
A. α1 receptor stimulation
B. α2 receptor stimulation
C. α1 receptor blockade
D. α2 receptor blockade
E. Nicotinic cholinergic stimulation
A

C. Priapism is defi ned as a persistent penile erection greater than 4 hours in duration,
which is unrelated to sexual stimulation or desire. Roughly 40 to 50% of patients who develop
priapism become impotent, even after surgical treatment. Drug-induced priapism accounts for
15–40% of all cases. Psychotropics associated with priapism include trazodone, phenothiazines,
butyrophenones, risperidone, and clozapine. Priapism results from decreased venous outfl ow
from the corpora cavernosa of the penis. This can be caused by obstruction of the venous system,
for example by blood dyscrasias such as sickle cell anaemia or by blocking the sympathetically
mediated (α1 receptor) detumescence. Hence the ability of a drug to block α1 receptors
correlates with its risk of priapism. This is especially true if the antiadrenergic effect is unopposed
by an equally strong anticholinergic effect. Sympathetic tone is related to detumescence while
parasympathetic tone is related to erection. For drugs with combined antiadrenergic and
anticholinergic activity, when antiadrenergic activity negates detumescence, the anticholinergic
activity will negate erection and so priapism will be rare.

How well did you know this?
1
Not at all
2
3
4
5
Perfectly
70
Q

Metacognitive abilities are proposed to be functions of the frontal lobe.
Metacognition refers to
A. Planning and sequential execution of motor acts
B. Ability to refl ect on one’s own cognitive processes
C. Problem-solving ability
D. Initiation and sustainment of motivation
E. Automatic cognitive processing without selected focus of attention

A

B. Metacognition refers to one’s knowledge concerning one’s own cognitive processes
and products of such processes. Metacognition is predominantly a function of prefrontal cortex.
Prefrontal damage leads to overestimation of abilities, lack of awareness of defi cits, and inability to
use feedback to change behaviour.

How well did you know this?
1
Not at all
2
3
4
5
Perfectly
71
Q

The ‘n-back test’ consists of making a response in accordance with a visual
or auditory stimulus presented ‘n’ items before the currently displayed
stimulus. This test is widely employed in neuroimaging paradigms primarily
to enable engagement of which of the following brain areas?
A. Frontal lobes
B. Occipital lobes
C. Cerebellum
D. Hippocampus
E. Amygdala

A

A. ‘N back’ test is a popular experimental paradigms for functional neuroimaging studies
of working memory. In this test subjects are asked to monitor a series of verbal or non-verbal
stimuli and to indicate when the currently presented stimulus is the same as the one presented
n trials previously. Using quantitative meta-analysis technique of normative functional imaging
studies, a broadly consistent activation of frontal and parietal cortical regions by various versions
of the n-back working memory paradigm has been demonstrated

How well did you know this?
1
Not at all
2
3
4
5
Perfectly
72
Q

Which of the following toxins has been used to simulate a model of
Parkinson’s disease?
A. Ketamine
B. Methyl phenyl tetrahydropyridine (MPTP)
C. Methylene dioxy methamphetamine (MDMA)
D. Vanillyl mandelic acid (VMA)
E. Hydroxy indole acetic acid (5HIAA)

A

B. MPTP occurred as an impurity when illicit synthesis of opioids was attempted by a
chemistry graduate student. He developed acute parkinsonian disease. Following detailed
investigations, animal models of Parkinson’s disease have been developed using MPTP as a
neurotoxin. VMA is a metabolite of epinephrine; 5HIAA is a metabolite of serotonin. MDMA
is the chemical name for ecstasy. Ketamine is a dissociative anaesthetic that stimulates sigma
receptors in brain. It is being increasingly used as a street drug.

How well did you know this?
1
Not at all
2
3
4
5
Perfectly
73
Q
How many layers are present in the laminar structure of the human
cerebral cortex?
A. Three
B. Four
C. Twelve
D. Six
E. Two
A

D. Human neocortex consists of a six-layered laminar structure. This cytoarchitectural
division has been largely adapted from Brodmann’s pioneering work. These six layers are
numbered from the top, that is the pial surface to the underlying white matter. In order, these are:
1. Molecular (or plexiform)
2. External granular layer
3. External pyramidal
4. Internal granular
5. Internal pyramidal (or ganglionic)
6. Multiform (or fusiform) layer.
The layers vary mostly in the size and density of pyramidal and stellate cells

How well did you know this?
1
Not at all
2
3
4
5
Perfectly
74
Q

Which of the following nuclei of the thalamus is primarily involved in the
relay of information for visual processing?
A. Supraoptic nucleus
B. Dorsomedial nucleus
C. Medial geniculate nucleus
D. Suprachiasmatic nucleus
E. Lateral geniculate nucleus

A

E. Lateral geniculate nucleus is the junction where axons of retinal ganglion cells terminate
after passing through uninterrupted via the optic nerve, optic chiasm, and optic tract. The medial
geniculate body is involved in auditory processing.

How well did you know this?
1
Not at all
2
3
4
5
Perfectly
75
Q
Which of the following cells are the only excitatory neurones in the
cerebellum?
A. Purkinje cells
B. Basket cells
C. Stellate cells
D. Granule cells
E. Golgi cells
A

D. Granule cells are the only excitatory neurones in the cerebellum. The cerebellar cortex
is a three-layered structure with the outermost layer containing two types of inhibitory neurones,
the stellate cells and basket cells. The middle layer has cell bodies of Purkinje cells (main output)
which are GABA-mediated and so are inhibitory in function. The innermost layer contains granule
cells, which are excitatory, and Golgi cells, which are inhibitory

How well did you know this?
1
Not at all
2
3
4
5
Perfectly
76
Q
Which of the following components of cognition is tested by the
digit span task?
A. Working memory
B. Implicit memory
C. Sensory memory
D. Autobiographic memory
E. Procedural memory
A

A. Working memory can be tested using digit span tasks. Using digit repetition forward,
a patient’s working memory capacity can be tested. Usually, a list of numbers (in no specifi c
pattern) is read aloud by the examiner and the patient is asked to repeat it immediately in same
order (forward span) or reverse order (backward span). Gradually, the length of the numeric
string is increased. Consistent error at a particular length is an indication for test termination. The
normal forward digit span is 7±2 for most adults. Backward span is more diffi cult and averages
around 5±2.

How well did you know this?
1
Not at all
2
3
4
5
Perfectly
77
Q

Ataxia can result from cerebellar lesions or posterior column lesions.
Though gait disturbances are predominant in both these conditions, which
of the following is seen in sensory ataxia but not cerebellar ataxia?
A. Nystagmus
B. Dysarthria
C. Loss of tendon refl exes
D. Absence of Romberg’s sign
E. Intact joint position sense

A

C. Sensory ataxia is due to posterior column disease, resulting from spinal cord lesions.
In this condition, loss of joint position sense and loss of tendon refl exes are seen. In cerebellar
ataxia, associated cerebellar signs such as dysarthria or nystagmus may be present. The corrective
effects of vision on balance and posture are seen in sensory ataxia. This is elicited by Romberg’s
test wherein swaying, which is almost absent when eyes are open and feet together, becomes
prominent on eye closure. In cerebellar ataxia, the patient may sway even with eyes open, which
worsens on eye closure

How well did you know this?
1
Not at all
2
3
4
5
Perfectly
78
Q

The ability of neurones to change the connection strength with other
neurones underlies the electrophysiological process called long-term
potentiation (LTP). Which of the following forms the neurochemical
basis of LTP?
A. Acetylcholine via nicotinic receptors
B. Substance P
C. Glutamate via NMDA receptors
D. Dopamine via D4 receptors
E. Cannabinoids via CB1 receptors

A

C. Long-term potentiation (LTP) is conceptualized as a more or less permanent increase
in synaptic effi cacy following high-frequency activity across the synapse. Glutamate via NMDA
receptor activation infl uences LTP. This may underlie changes in synaptic plasticity observed in
learning- and memory-related processes. LTP is proposed to be the cellular biological correlate
of long-term memory.

How well did you know this?
1
Not at all
2
3
4
5
Perfectly
79
Q
Which of the following is a major dopaminergic site?
A. Nucleus basalis
B. Ventral tegmental area
C. Dorsal raphe nucleus
D. Spinal interneurones
E. Locus coeruleus
A

B. Most neurones from the ventral tegmental area of midbrain ascend in the medial
forebrain bundle and the nigrostriatal pathway. These neurones are rich in dopamine. The
dopamine neurones of the ventral tegmental area (VTA) are thought to play a central role
in reward, motivation, and drug addiction. Nucleus basalis of Meynert is a major cholinergic
site. Dorsal raphe nucleus is predominantly a serotonergic site. Locus coeruleus contains
noradrenergic neurones

How well did you know this?
1
Not at all
2
3
4
5
Perfectly
80
Q
Which of the following brain regions shows a preferential degeneration in
Alzheimer’s disease?
A. Nucleus basalis
B. Ventral tegmental area
C. Dorsal raphe nucleus
D. Spinal interneurones
E. Locus coeruleus
A

A. Nucleus basalis of Meynert contains a majority of cholinergic neurones. Apart from
hippocampal (entorhinal cortex) neuronal loss, selective loss of neurones in the nucleus basalis
has led to the pursuit of cholinergic theories of memory impairment in dementia. Currently
available pharmacological interventions largely target cholinergic defi ciency in Alzheimer’s
disease.

How well did you know this?
1
Not at all
2
3
4
5
Perfectly
81
Q
Processing of fear conditioning is associated with functions of the
A. Planum temporale
B. Heschl’s gyrus
C. Amygdala
D. Anterior pituitary
E. Angular gyrus
A

C. SM, a patient with rare bilateral amygdala damage, was initially reported to lack the ability
to recognize fear from facial expressions. Since the report of her case, a number of lesion and
functional imaging studies have demonstrated the role of the amygdala in fear processing. It is
possible that the amygdala mediates spontaneous fi xations on the eyes when viewing expressions
of faces. Lack of such fi xation may lead to failure of spontaneous processing of fearful emotions

How well did you know this?
1
Not at all
2
3
4
5
Perfectly
82
Q

Which of the following enzymes involved in neurotransmitter synthesis is
directly affected by pyridoxine defi ciency?
A. Glutamate decarboxylase
B. Acetyl cholinesterase
C. Dopamine hydroxylase
D. Tyrosine hydroxylase
E. Tryptophan hydroxylase

A

A. Pyridoxine (vitamin B6) when phosphorylated to pyridoxal phosphate acts as a coenzyme
in the conversion of glutamic acid to GABA, mediated by the rate-limiting enzyme glutamate
decarboxylase. The pivotal role of this chemical interaction is evident from pyridoxine-dependent
seizures, which can occur in relation to mutations of chromosome 5q31. Dysfunction of this
enzyme leads to glutamate accumulation and excitatory damage via NMDA receptors

How well did you know this?
1
Not at all
2
3
4
5
Perfectly
83
Q
Which of the following brain areas has a relatively permeable blood–tissue
interface?
A. Anterior pituitary
B. Hippocampus
C. Subfornicular organ
D. Cerebellum
E. Lateral surface of frontal lobes
A

C. The circumventricular organs are midline structures around the third and fourth
ventricles. Pineal gland, median eminence, neurohypophysis, subfornical organ, area postrema,
subcommissural organ, organum vasculosum of the lamina terminalis, and the choroid plexus
are considered as circumventricular organs. These structures lack the blood–brain barrier seen
in other regions of brain. These areas enable the brain’s direct response to chemical challenges
in blood.

How well did you know this?
1
Not at all
2
3
4
5
Perfectly
84
Q

Which of the following terms refers to a substance that infl uences neuronal
activity and originates from non-synaptic sites?
A. Neurotransmitter
B. Neurotrophin
C. Neuromodulator
D. Second messenger
E. Neurohormone

A

C. A neuromodulator is a substance that enhances or diminishes the effect of
neurotransmitters but does not usually result in neuronal conduction changes on its own.
Substance P, enkephalin, cholecystokinin, somatostatin, and neuropeptide Y are examples
of neuromodulators. Neurotrophin is a substance produced to infl uence neuronal growth.
Neurohormones are substances released by neurones into the blood stream to infl uence
effector organs at distant sites, for example corticotrophin-releasing hormone.

How well did you know this?
1
Not at all
2
3
4
5
Perfectly
85
Q
Which of the following neurotransmitters act as a physiological antagonist
for acetylcholine?
A. Serotonin
B. Substance P
C. Neurokinin
D. Norepinephrine
E. Nicotine
A

D. Norepinephrine acts as physiological antagonist for acetylcholine. Physiological
antagonism is defi ned as the process wherein two chemical molecules that act through two
different receptor systems result in opposing actions in the body that tend to negate each other’s
physiological effect. Insulin and glucagon can be considered as physiological antagonists, to some
extent.

How well did you know this?
1
Not at all
2
3
4
5
Perfectly
86
Q
Which of the following is not a ligand-gated ion channel?
A. Nicotinic cholinergic receptors
B. GABAA receptors
C. Glycine receptors
D. NMDA receptors
E. Muscarinic cholinergic receptors
A

E. Muscarinic receptors act via the G protein-coupled second messenger system. Nicotinic
cholinergic receptors operate via ligand gated channels that are permeable to Na+, K+ and
sometimes Ca2+. GABAA receptors are also ligand-gated ion channels that allow Cl– ions to pass
through, resulting in inhibitory activity. Glycine receptors are strychnine-sensitive ligand-gated
ion channels with inhibitory activity. Metabotropic receptors aside, most glutamate receptors
(NMDA, AMPA, and kainate) are ionotropic and allow Ca2+ transit (NMDA, AMPA) or Na+, K+
transit (kainate) via ligand-gated channels

How well did you know this?
1
Not at all
2
3
4
5
Perfectly
87
Q

11C raclopride is a commonly used PET ligand. Which of the following
receptors can be identifi ed by 11C raclopride?
A. Serotonin receptors
B. GABAA receptors
C. Muscarinic cholinergic recerptors
D. Postsynaptic dopamine receptors (D2/D3)
E. Presynaptic dopamine reuptake inhibitor

A

D. 11C raclopride is used to identify postsynaptic dopamine receptors (D2/D3). Raclopride
is related to the antipsychotic sulpride and amisulpride (substituted benzamide); hence it acts as a
D2/D3 inhibitor

88
Q

Leptin is a hormone involved in regulation of adiposity and meal intake.
Which of the following structures is the major source of leptin?
A. Hypothalamus
B. Gut epithelium
C. Hepatocytes in liver
D. Adipose tissue
E. Mitochondria-rich muscle fi bres

A

D. Adipose tissues are the major source of leptin. The amount of leptin in plasma is directly
proportional to degree of adiposity in one’s body. Leptin enters the brain, by an unknown
mechanism, to reduce appetite via a central mechanism. In obesity, a certain degree of leptin
resistance exists, leading to high circulating levels of leptin

89
Q
Which of the following substances does not interact with cell membrane
receptors and acts directly on nuclear material to produce physiological
effects?
A. Serotonin
B. Growth hormone
C. Norepinephrine
D. Dopamine
E. Thyroid hormone
A

E. Thyroid hormone acts in a similar manner to steroid hormone. Receptors for these
hormones are not present in the cell membrane but, upon cellular entry, they bind to nuclear
receptors to directly affect cellular processes. These nuclear receptors act as hormone-activated
transcription factors that modulate gene expression

90
Q

A 77-year-old woman dies of severe pneumonia with sepsis at a care
home. She had poor episodic memory for recent events and deteriorating
language functions for at least 2 years before her death. The most likely
pathology that could be seen in her brain tissue is
A. Ballooning of cells with intraneuronal accumulations
B. Diffuse and neuritic plaques with amyloid deposits in cortex and hippocampus
C. Diffuse amyloid deposition restricted to blood vessels
D. Lysosomal accumulations within neuronal cytoplasm
E. Eosinophilic rod-shaped inclusions of actin fi laments

A

B. The patient described in the question is most likely to have a diagnosis of Alzheimer’s
dementia. Diffuse and neuritic plaques with amyloid deposits in cortex and hippocampus are
highly suggestive of Alzheimer’s disease. Ballooning of cells with intraneuronal accumulations
suggests Pick’s disease. Diffuse amyloid deposition restricted to blood vessels is seen in amyloid
angiopathy. Eosinophilic rod-shaped inclusions of actin fi laments are Hirano bodies, seen
sometimes in Alzheimer’s disease

91
Q
Which of the following receptors, on stimulation, reduces serotonin release?
A. 5-HT1B
B. 5-HT1D
C. 5-HT1A
D. 5-HT2
E. 5-HT3
A

A. 5-HT1B receptors are present as presynaptic autoreceptors. On stimulation, these
receptors can decrease further serotonin release. Postsynaptic 5-HT1B receptors may have a role
in locomotor activity and aggression. 5-HT1A receptors are associated with antidepressant activity.
5-HT2A receptors are implicated in hallucinogenic properties of LSD-like drugs and blockade is
associated with antipsychotic activity. 5-HT3 is the only ligand-gated ion channel among serotonin
receptors; it is present in gut in large numbers and may mediate emesis

92
Q

All of the following are true with respect to dopamine metabolism except
A. Monoamine oxidase is the primary metabolic enzyme.
B. All dopamine from the re-uptake process gets repackaged into presynaptic vesicles.
C. Catechol-O-methyl transferase is present in cytoplasm of postsynaptic neurones.
D. Homovanillic acid is a primary metabolite.
E. Monoamine oxidase type B is more selective for dopamine

A

B. Dopamine exerts its action via one of two families of dopamine receptors. After receptor
stimulation, dopamine can either be taken back into the presynaptic neurone and destroyed
or repackaged as a neurotransmitter. Among those dopamines that are taken back, some are
metabolized by monoamine oxidase (MAO). MAOB selectively metabolizes dopamine.
Catechol-O-methyltransferase (COMT) is localized in the postsynaptic neuronal cytoplasm
and is involved in secondary metabolism of dopamine. The primary metabolite of dopamine
is homovanillic acid (HVA)

93
Q

Neuroendocrine abnormalities have been documented in depression and
other psychiatric disorders. Which of the following is not a neuroendocrine
abnormality seen in depression?
A. Blunted growth hormone (GH) response to levodopa
B. Enhanced growth hormone response to apomorphine
C. Blunted thyroid-stimulating hormone (TSH) response to thyrotrophin-releasing
hormone (TRH)
D. Failure of exogenous steroid to suppress cortisol
E. Blunted prolactin release on clomipramine challenge

A

B. In depression, hypercortisolism is seen in a subgroup of patients. This can be
demonstrated by the dexamethasone suppression test wherein exogenously administered steroid
dexamethasone fails to activate the negative feedback loop to reduce endogenous cortisol.
Depressed patients also show blunted TSH response when thyrotrophin-releasing hormone
(TRH) is injected. A brisk GH release seen on apomorphine/levodopa challenge is absent in some
patients with depression. This suggests dopaminergic dysfunction in a subgroup. Clomipramine
challenge in normal subjects leads to increased serotonergic activation and prolactin release. This
prolactin response to clomipramine-mediated via serotonin is blunted in depression. Psychotic
depression shows more HPA axis disturbance than non-psychotic depression. Melancholic
(somatic syndrome) depression increases the chance of fi nding endocrine disturbances among
depressed patients.

94
Q

Event-related potentials (ERP) are records of the brain’s electrical activity in
response to various stimuli. P300 is an extensively studied ERP in psychiatry.
Which of the following is true with respect to P300 abnormalities in
schizophrenia?
A. P300 is a state rather than a trait marker in schizophrenia.
B. Schizophrenia with early age of onset shows less pronounced P300 abnormalities.
C. Reduced P300 amplitude is specifi c for schizophrenia.
D. Paranoid subtype is associated with more pronounced P300 abnormalities.
E. P300 is affected by duration of disease in patients with schizophrenia.

A

D. P300 is a positive component that occurs 250–500 ms after stimulus onset. Amplitude
and latency of P300 is altered in schizophrenia. These changes refl ect a trait rather than being
related to psychotic state, but these are not specifi c to schizophrenia; for instance Alzheimer’s
disease patients show similar defi cits. The reduction in amplitude is more pronounced in earlieronset
and paranoid subgroups. P300 is not affected by medication status, disease duration, or
severity of psychopathology.

95
Q

Which of the following tests for intelligence is not infl uenced by language or
formal education?
A. Weschler’s Adult Intelligence Scale
B. Stanford–Binet Test
C. Raven’s Progressive Matrices
D. Weschler’s Intelligence Scale For Children
E. Kaufman Adolescents and Adults Intelligence Test

A

C. Fluid intelligence includes non-verbal intellectual functions such as problem solving
and reasoning; these are not infl uenced by one’s cultural experience or education. Crystallized
intelligence includes acquired knowledge, which is heavily infl uenced by culture, formal education,
and opportunities. Kaufman Adolescent and Adult Intelligence Test measures both ‘fl uid’ and
‘crystallized’ intelligence. The Wechsler Intelligence Scale for Children and the Wechsler Adult
Intelligence Scale measure IQ and are infl uenced by cultural differences and formal educational
attainment. Stanford–Binet Scale is one of the earliest intelligence scales and is infl uenced by
school education. Raven’s Progressive Matrix is a measure of ability to form perceptual relations
and analogical reason; it is independent of language and formal schooling. It can be used for
anyone above age 6.

96
Q

Neuropeptides such as endorphins are produced from the degradation of a
large precursor molecule called pro-opiomelanocortin (POMC). Which of
the following hormones is synthesized from the same precursor?
A. Thyroxine
B. Thyroid-stimulating hormone (TSH)
C. Adrenocorticotrophic hormone (ACTH)
D. Growth hormone
E. Antidiuretic hormone (ADH)

A

C. Pro-opiomelanocortin (POMC) is a polypeptide precursor protein. It produces many
biologically active peptides including the melanocyte-stimulating hormones (MSH), corticotrophin
(ACTH), and β-endorphin. MSH and ACTH are collectively known as melanocortins and a family
of specifi c receptors has been described for them. In the CNS, POMC-producing neurones are
located in the arcuate nucleus of the hypothalamus and the nucleus tractus solitarius of the
brainstem. The CNS POMC system regulates feeding behaviour, sexual behaviour, lactation, the
reproductive cycle, and possibly central cardiovascular control.

97
Q
All of the following physiological effects are mediated by GABA
potentiation except
A. Sedation
B. Working memory
C. Increase in seizure threshold
D. Muscle relaxation
E. Inhibition of memory consolidation
A

B. GABA does not directly mediate working memory. Prefrontal dopamine via D1 receptors
is thought to heavily infl uence working memory capacity. GABAA activation has sedative
properties. GABAA activation leads to an increase in seizure threshold via inhibitory activity.
GABAA also plays a role in memory consolidation. GABA agonists prevent formation of long-term
memory, though the process of learning itself may not be affected. GABAB activation mediates
central muscle relaxation via spinal cord and brain receptors

98
Q

Which of the following structures in human brain show morphological
differences according to one’s sexual orientation?
A. Red nucleus
B. Medial dorsal thalamus
C. Nucleus tractus solitarius
D. Interstitial nucleus of anterior hypothalamus
E. Pineal gland

A

D. Several neuroanatomical differences between homosexuals and heterosexuals have been
reported from human studies. The suprachiasmatic nucleus (SCN) has been noted to be larger
and more elongated in homosexual males (and females in general). The third interstitial nucleus
of the anterior hypothalamus (INAH-3) was noted to be smaller among homosexuals. Anterior
commissure is larger in homosexual men than in heterosexual men. No sexual-orientation
differences have been reported in the sexually dimorphic nuclei of the preoptic area in humans.
LeVay S.

99
Q

Which of the following proteins is the major component of Lewy bodies
seen in Parkinson’s disease and Lewy body dementia?
A. Amyloid precursor protein
B. Prion protein
C. Ceruloplasmin
D. Amylin
E. Alpha synuclein

A

E. Lewy bodies contain ubiquitin (non-specifi c) and alpha synuclein. Synucleiopathies refer
to a set of disorders in which aggregates of synculein accumulation is noted. Lewy bodies are
demonstrated pathologically in these disorders, but unlike the Lewy bodies in the substantia
nigra seen in Parkinson’s disease, cortical Lewy bodies are not surrounded by a distinct halo
that enables their identifi cation. Recently, the development of immunostaining for ubiquitin and
synuclein have increased the accuracy of visualizing Lewy bodies

100
Q
Proliferation of large protoplasmic astrocytes results in atypical cells
such as Opalski cells and Alzheimer cells. Presence of such astrocytes is a
pathological feature of
A. Alzheimer’s dementia
B. Fragile-X syndrome
C. Frontotemporal dementia
D. Multiple sclerosis
E. Wilson’s disease
A

E. Wilson’s disease is also known as hepatolenticular degeneration. It is an autosomal
recessive disorder of copper metabolism. The abnormal gene (ATP7B) is located in chromosome 13.
Ceruloplasmin is an α2 globulin that normally carries 90% of the copper present in the plasma.
In Wilson’s disease, ceruloplasmin fails to bind copper and its excretion by the liver is impaired.
The excess copper accumulates fi rst in the liver and then in the brain (especially putamen and
pallidus) and other tissues. In the early stages of the disease, proliferation of large protoplasmic
astrocytes, such as Opalski cells and Alzheimer cells, occurs. Deposition of copper in Descemet’s
membrane in the cornea leads to the appearance of the golden-brown ‘Kayser–Fleischer’ (KF)
ring. The main neurological abnormalities are rigidity, dystonia, chorea, athetosis, dysarthria, and
tremor. It is considered that a pure psychiatric presentation occurs in 20–25% of cases. Around
50% of patients will have mental disturbances at some point during the course of the disease.
Psychiatric manifestations tend to occur more commonly with neurological forms of Wilson’s
disease than with the hepatic form of the clinical syndrome. Cognitive impairment occurs in
up to 25% of patients. The dementia is usually subcortical type with frontal defi cits. Depression
occurs in 30% of cases and suicidal behaviour may occur in between 4 and 16%.

101
Q

Which of the following statements regarding the teratogenic effect of
alcohol on neuronal development is false?
A. Alcohol induces apoptosis in the fetal brain.
B. Even a single episode of excessive alcohol exposure can be harmful.
C. The most prominent pathological fi nding in fetal alcohol syndrome is scarring of the
thalamus.
D. Facial malformations occur when the exposure is in the fi rst trimester.
E. Thiamine defi ciency is the most important mediator of neuronal damage seen in fetal
alcohol syndrome.

A

E. It has been postulated that exposure of the brain to alcohol during the period of rapid
synaptogenesis (third trimester) leads to apoptosis and neuronal loss, especially in thalamic
and basal ganglia area. Since this change is apoptotic, it does not lead to fi brous scarring. This
apoptotic cell death has been postulated to be due to the action of alcohol on the NMDA and
GABA receptors. Apparently, large quantities of alcohol taken during a short period of time, even
once, as in a binge episode, can lead to these changes. The cluster of symptoms also depends on
the timing of alcohol exposure, so if the exposure occurred during the fi rst trimester it would
lead to facial abnormalities. In addition to developing hyperactivity/attention defi cit disorder
and varying degrees of learning impairment in children, a high percentage have adult-onset
neuropsychiatric disturbances, including major depression and psychosis. There is no evidence to
suggest that thiamine defi ciency mediates neuronal damage in fetal alcohol syndrome

102
Q
Features of the cauda equina syndrome include all except
A. Asymmetric leg weakness
B. Loss of deep tendon refl ex
C. Prominent sphincter dysfunction
D. Low back pain
E. Sensory loss
A

C. Cauda equina syndrome is characterized by low back pain (radicular pain), asymmetric
weakness, arefl exia, and sensory loss in the legs, and relative sparing of bowel and bladder
function. The syndrome usually results from injury of multiple lumbosacral nerve roots within
the spinal canal. Most commonly, this is due to a ruptured lumbosacral intervertebral disc (disc
prolapse). Less common causes include lumbosacral spine fracture, haematoma within the spinal
canal, and compressive tumours. Cauda equine must be differentiated from conus medullaris
syndrome, which includes compression of the lower sacral and coccygeal segments. This results in
bilateral saddle anaesthesia, prominent bladder and bowel dysfunction, and impotence. Cutaneous
refl exes are absent, but the muscle strength is usually preserved in conus medullaris syndrome.
Kasper DL

103
Q
Which of the embryonic germ cell layers give rise to neurones?
A. Neural tube
B. Neural crest
C. Mesoderm
D. Ectoderm
E. Notochord
A

D. During embryonic development, gastrulation leads to the formation of the germ
layers—ectoderm, mesoderm, and endoderm. Skin and nervous system develops from the
ectoderm, the gut from the endoderm, and all other visceral organs from the mesoderm. After
gastrulation, the notochord is formed from mesodermal cells. This notochord lies rostro caudally
and it sends signals to the ectoderm lying adjacent to it, transforming them into neurectoderm.
This forms the neuronal stem cells in the embryo. This strip of ectoderm is otherwise called the
neuronal plate. By the third week of gestation, a groove develops on the dorsal aspect of the
neural plate and this groove gradually deepens, forming neuronal folds. These folds gradually close
bidirectionally by the end of the fourth week, forming the neural tube. Closure of the neural tube
is susceptible to teratogenic infl uences operating in fi rst trimester (e.g. valproate).

104
Q
All of the following are features of lesion to the cerebellum except
A. Dysarthria
B. Dysmetria
C. Intentional tremor
D. Ipsilateral hypotonicity
E. Sphincter disturbance
A

E. Sphincter disturbance is not a feature of isolated cerebellar lesions. Unilateral lesions
of the cerebellum affect ipsilateral limbs. Symptoms of cerebellar damage include asynergia (loss
of coordination between muscles leading to jerky movements), dysmetria (defects in reaching
targets via crude motor movements, for example past pointing on fi nger–nose test), intention
tremor, dysarthria, disturbed balance including gait ataxia (produces wide-based, staggering
gait prone to shuffl e or fall), hypotonia, dysdiadochokinesia (inability to do rapid alternate
movements), and nystagmus. The cerebellum has been suggested to be a seat of cognitive
function, especially working memory

105
Q
Which of the following cranial nerves is purely afferent?
A. Facial nerve
B. Glossopharyngeal nerve
C. Hypoglossal nerve
D. Trochlear nerve
E. Vestibulocochlear nerve
A

E. The vestibulocochlear nerve is purely sensory. It has two components, the vestibular
and the cochlear, both of which are sensory. The vestibular component transmits information
on position and balance received from the semicircular canals and the cochlear component
serves the sense of hearing. The hypoglossal nerve innervates the ipsilateral side of the tongue.
It is a purely motor efferent nerve. In unilateral lower motor neurone palsy of the eight nerve,
when protruded, the tongue deviates toward the side of weakness. The trochlear nerve is unique
among cranial nerves in that it decussates to the contralateral side and its point of exit is through
the dorsal surface of the brain. The trochlear nerve thus innervates the superior oblique muscle
of the contralateral eye. It is a purely efferent nerve. The facial nerve innervates the muscles of
the face. It has a sensory component, innervating the anterior two-thirds of the tongue, via the
chorda tympani. Facial nerve palsy causes deviation of the angle of mouth to the normal side.
As a mnemonic, ‘the rule of 17’ applies to deviations in cranial nerve palsies. Palsy of tenth (vagus
and hence palate) and seventh (facial) nerve leads to deviation to the normal side. Palsy of the
fi fth (trigeminal and hence the jaw muscles) and twelfth (hypoglossal, tongue) nerve leads to
deviation to the paralytic side.

106
Q
The light refl ex pathway includes all of the following except
A. Ciliary muscles
B. Edinger–Westphal nucleus
C. Occipital cortex
D. Occulomotor nerve
E. Optic nerve
A

C. The pathway that enables normal vision starts from rods and cones in the retina, which
are receptors of the ganglion cells. The axons of ganglion cells extend as the optic nerve, through
the optic chiasma, via the optic tract, and synapse at the lateral geniculate body of thalamus.
From the lateral geniculate body, second-order neurones go through the optic radiation to the
visual cortex in the occipital lobe. In contrast, the pathway constituting the papillary light refl ex
digresses from the visual pathway before it joins the lateral geniculate body to reach the dorsal
midbrain. They synapse at the pretectal nuclei, from where second-order neurones go to the
Edinger–Westphal nucleus on both sides. Edinger–Westphal nuclei, via the third nerve, control
the pupillary constrictors that constitute the response to light. So, pupillary light refl ex do not
involve the occipital cortex. Hence even in those with cortical blindness due to bilateral occipital
cortex damage, light refl ex may be intact

107
Q
All of the following carry parasympathetic fi bres in them except
A. Facial nerve
B. Glossopharyngeal nerve
C. Hypoglossal nerve
D. Oculomotor nerve
E. Vagus nerve
A

C. The hypoglossal nerve is a purely somatic motor nerve. The autonomic nervous
system has two parts, the sympathetic and parasympathetic. Sympathetic output from the
CNS is mainly through thoracic and lumbar spinal nerves. Sympathetic preganglionic nerves
are short and form synapses in paired ganglia adjacent to the spinal cord. The parasympathetic
system has a craniosacral output, that is it operates through some cranial nerves and sacral
spinal nerves. These have long preganglionic nerves which form synapses at ganglia near or on
the organ innervated. Among the cranial nerves, the vagus is the chief parasympathetic nerve.
It supplies parasympathetic efferents to heart and most of the abdominal viscera and the
gastrointestinal tract, but oculomotor (III), facial (VII), and glossopharyngeal nerves (IX) also carry
parasympathetic fi bres. The neurotransmitter at the preganglionic nerve ending is acetylcholine
in both sympathetic and parasympathetic systems. At the post ganglionic nerve ending, the
neurotransmitter is acetylcholine in the parasympathetic system and mostly norepinephrine in
the sympathetic system.

108
Q
Which of the following is a result of parasympathetic activity?
A. Decreased salivation
B. Ejaculation
C. Increased heart rate
D. Relaxation of external anal sphincter
E. Relaxation of the ciliary muscles
A

D. The sympathetic system is responsible for the ‘fl ight and fi ght’ and the parasympathetic
system for the ‘rest and digest’ reactions. Generally, they are considered to have opposing actions.
The sympathetic system is activated in emergency situations, where the body requires more
energy. This response includes increased cardiac output, dilatation of bronchioles, routing blood
to the muscles, glycogen and fat breakdown leading to a rise in the blood glucose and fatty acids,
and slowing down of digestion and renal fi ltration. This also leads to a decrease in gastrointestinal
secretion and motility (leading to dryness of the mouth). In addition, sympathetic activity causes
constriction of bladder and bowel sphincters and relaxation of the smooth muscles of the
viscera. The pupils dilate due to action on the dilators. In contrast, parasympathetic stimulation
leads to pupillary constriction and accommodation for close vision, reduces heart rate,
constricts bronchioles, and increases gastrointestinal secretions with relaxation of sphincters.
Parasympathetic stimulation is necessary for erection and sympathetic stimulation for ejaculation.
Ganong WF.

109
Q
The posterior column of the spinal cord is responsible for all of the
following except
A. Light touch
B. Proprioception
C. Tactile localization
D. Temperature discrimination
E. Vibration sensation
A

D. The posterior column is responsible for transmission of proprioception, light touch,
tactile localization, and vibration senses. Posterior column dysfunction can result in disturbances
in the knowledge of extremity movement and position. This presents as sensory ataxia (noted
fi rst in the dark as visual input does not compensate for the lost position sense) and a positive
Romberg’s sign. Pain and temperature is transmitted to the central nervous system through the
spinothalamic tract. Spinothalamic tracts cross over two segments above the level of entry of
the root at the spinal cord. Posterior column tracts cross over only at the midbrain level, where
they synapse with the cuneate and gracilis nuclei. Hence, if hemisection of the cord takes place,
ipsilateral posterior column senses are lost below the level of section; contralateral spinothalamic
sensations are lost from two levels below the site of section.

110
Q
Which of the following is not a component of the basal ganglia?
A. Amygdala
B. Caudate nucleus
C. Globus pallidus
D. Substantia nigra
E. Subthalamic nucleus
A

A. The term basal ganglia traditionally applied to fi ve large, subcortical nuclear masses on
each side of the brain: the caudate nucleus, putamen, and globus pallidus, subthalamic nucleus, and
substantia nigra. The globus pallidus is further divided into an external and an internal segment,
and the substantia nigra is divided into a pars compacta and a pars reticulata. The caudate
nucleus and the putamen are frequently called the striatum; the putamen and the globus pallidus
are sometimes called the lenticular nucleus. The nucleus accumbens is the region where the
putamen and the caudate merge anteriorly. What structures comprise the basal ganglia has been
a debate over the years. More recently, an additional term, ‘ventral striatum’ has been introduced
to describe those parts of the striatum (caudate and putamen) closest to limbic structures and
that are involved in cognitive and behavioural functions. The ventral striatum includes the nucleus
accumbens, which plays a major role in motivational and reward-related behaviour. Amygdala is
closely related to the basal ganglia due to its functional and structural proximity. The amygdala
complex develops from the same tissue mass as the caudate nucleus

111
Q

Nystagmus is a recognized feature of all of the following except
A. Barbiturate toxicity
B. Cerebellopontine angle tumours
C. Horner’s syndrome
D. Multiple sclerosis
E. Vertibrobasillar artery insuffi ciency

A

C. Horner’s syndrome is caused by sympathetic dysfunction at the craniocervical output.
Clinical features include ipsilateral mild (usually <2 mm) ptosis, anisocoria (unequal pupils)
due to ipsilateral miosis, enophthalmos, loss of ciliospinal refl ex, and ipsilateral facial anhidrosis
(mnemonic ‘PAMELA’). Nystagmus is generally not seen. Nystagmus is a rhythmical oscillation
of the eyes, occurring pathologically in a wide variety of diseases. Abnormalities of the eyes or
optic nerves, especially when the onset is in childhood, may present with nystagmus (pendular
or jerk nystagmus). Jerk nystagmus is characterized by a slow drift off the target, followed
by a fast corrective saccade. Jerk nystagmus can be downbeat, upbeat, or horizontal (left or
right) with the names being given according to the direction of the fast phase. Gaze-evoked
nystagmus is the most common form of jerk nystagmus, where the subject is asked to look to
the corner of the eye. Exaggerated gaze-evoked nystagmus can be seen in: drug intake/toxicity
(sedatives, anticonvulsants, alcohol); muscle paresis; myasthenia gravis; demyelinating disease; and
cerebellopontine angle, brainstem, and cerebellar lesions. Downbeat nystagmus usually occurs
from lesions near the craniocervical junction, while upbeat nystagmus is associated with damage
to the pontine tegmentum, from stroke, demyelination, or tumour. Vestibular system dysfunction
also leads to nystagmus. This occurs in discrete attacks, usually associated with sudden
movements of the head and is accompanied by symptoms of nausea, tinnitus, hearing loss, and
vertigo.

112
Q

Which of the following is a feature of normal ageing?
A. Preserved immediate memory
B. Decline in general intelligence measures
C. Preserved executive abilities such as abstraction
D. Intact verbal fl uency
E. Intact cognitive processing speed

A

A. Ageing results in declines in a variety of cognitive domains, but some abilities appear to
be relatively preserved. The relatively unaffected faculties include general intellectual knowledge,
vocabulary and comprehension, attention processes, language abilities related to phonologic,
lexical, and syntactic knowledge, motor skills that are learned early in life and repeatedly used,
and immediate and implicit memory, including some aspects of short-term memory. Signifi cant
age-associated decline is seen in processing speed, ability to reason and solve problems, fl uid
intelligence, dividing attention between two tasks, executive function domains (mental fl exibility,
abstraction, and concept formation), visuospatial skills (drawing, construction, and maze learning),
language skills involving semantic knowledge needed for naming and verbal fl uency, motor skills
that require speed, and ability to learn and retain new information for long-term access

113
Q
Which of the following lesions is commonly associated with gelastic
(laughing) seizures?
A. Hypothalamic tumours
B. Hippocampal sclerosis
C. Occipital lesions
D. Inferior parietal damage
E. Lesions of corpus callosum
A

A. Gelastic seizures are characterized by shallow laughter occurring in fi ts. The laughter
usually lasts less than 1 minute and is then followed by signs of complex partial or focal seizures,
such as eye and head movement, automatisms such as lip-smacking, and altered awareness. In
many cases, these associated features may be absent, resulting in delayed diagnosis. The most
common areas of the brain associated with gelastic seizures are the hypothalamus, the temporal
lobes, and the frontal lobes. A combination of gelastic seizures and precocious puberty is often
noted and can be attributed to hamartoma of the hypothalamus

114
Q
Which of the following is a ligand-gated ion channel?
A. Muscarinic receptor
B. Norepinephrine receptor
C. Nicotinic receptor
D. GABAB receptor
E. Metabotropic glutamate receptor
A

C. The cholinergic nicotinic receptor forms the prototypical model for ionotropic
receptors. It is a heteromeric pentameric protein, and each subunit is a transmembrane protein
with four transmembrane domains (in total 20 transmembrane domains compared to seven in
the case of metabotropic receptors). Binding of a neurotransmitter to the extracellular domain
of the ionic receptor results in the brief opening of a transmembrane ionic pore. This leads to an
infl ux of certain ions, which produces a brief modifi cation of the resting membrane potential. This
results in a postsynaptic action potential. Muscarinic acetyl choline receptors are not ionotropic;
they are G-protein coupled and act via second messengers. GABAA not GABAB is an ion channel
receptor. Norepinephrine receptors are largely G-protein coupled; they are not ion channels.

115
Q

A 32-year-old man presents with progressive muscle weakness, cognitive
impairment, and involuntary movements of upper limbs and facial muscles.
A blood fi lm reveals spiked red blood cells. He is troubled by compulsive
behaviour pertaining to order and symmetry. Which of the following
conditions is most likely?
A. Acanthocytosis
B. Huntington’s chorea
C. Haemophilia A
D. Wilson’s disease
E. Progressive supranuclear palsy

A

A. The presence of spiked red blood cells (RBCs) together with ataxia, progressive
weakness, and cognitive impairment is suggestive of neuroacanthocytosis. Patients with
neuroacanthocytosis may also show personality changes characterized by impulsivity,
distractibility, and compulsivity. Neuropathological fi ndings include severe atrophy of the caudate
and putamen with loss of neurones and an associated astrocytic reaction. Less severe changes
are seen in the pallidum, thalamus, substantia nigra, and anterior horn cells of spinal cord.
Acanthocytes are spiked RBCs seen in peripheral blood smears. Acanthocytosis is also seen in
patients with abetalipoproteinaemia or hypobetalipoproteinaemia, where serum vitamin E and
lipoprotein levels are abnormal.

116
Q

The recreational drug LSD exerts its hallucinogenic effect as a partial
agonist at which of the following receptors?
A. Dopamine D1
B. Glutamate AMPA
C. Serotonin 5-HT1A
D. Serotonin 5-HT2A
E. Cannabinoid CB1

A

D. The exact mechanisms of action of LSD and other hallucinogens are not completely
understood as yet, but there is substantial evidence pointing towards serotonergic systems in the
brain. Receptor-binding studies have shown that radiolabelled LSD binds to 5-HT2A and 5-HT1C
receptors. Hallucinogens have agonist actions at the 5-HT2A receptor. The psychoactive and
behavioural effects of hallucinogens are blocked by 5-HT2A antagonists. Tolerance/tachyphylaxis
of hallucinogenic effect is related to down-regulation of 5-HT2A receptors. There may be a role
for 5-HT2C receptors too in mediating the actions of hallucinogens. LSD is only a partial agonist
at 5-HT2A, in contrast to the full agonist actions of other hallucinogens. The 5-HT2A receptor
potentiates glutamatergic and dopaminergic neurotransmission when activated, while activating
the inhibitory GABA interneurone system

117
Q
Which of the following neurotransmitters is implicated in the
neurobiology of addiction and behaviours associated with craving seen in
recreational drug users?
A. GABA
B. Dopamine
C. Substance P
D. Serotonin
E. Neurosteroids
A

B. The acute administration of all addictive drugs (except benzodiazepines) stimulates
dopamine transmission in the projection from the ventral mesencephalon to the nucleus
accumbens. This projection is generally referred to as the mesolimbic dopamine system. The site
of action by which different street drugs activate dopamine can be classifi ed into three distinct
types:
1. Increasing the presynaptic release of dopamine without directly altering the activity of
dopamine neurones, for example stimulants acting via dopamine reuptake channels
2 Stimulation of dopamine neurones via receptors on dopaminergic neuronal membrane, for
example marijuana via cannabinoid receptors
3. Via decrease of inhibitory input into dopaminergic cells leading to disinhibition of dopamine
activity, for example opioids and alcohol

118
Q
Which of the following can be classifi ed as a neurotrophin?
A. BDNF
B. Protein kinase C
C. Nitric oxide
D. COMT
E. Vasopressin
A

A. Neurotrophins comprise a family of proteins including nerve growth factor (NGF),
brain-derived neurotrophic factor (BDNF), and neurotrophins (NT)-3, -4/5, and -6.
Proneurotrophins are enzymatically processed to create mature neurotrophins. Neurotrophins
bind to specifi c tyrosine kinase receptors. Neurotrophins promote neuronal growth,
differentiation, and survival, and modulate synaptic plasticity. These growth-related effects result
from the interaction of neurotrophins with mitogen-activated protein kinase (MAPK) signalling
pathway and the phosphatidylinositol-3 kinase pathway. In addition, the neurotrophins can inhibit
cell death cascades. Reduced expression of neurotrophins such as BDNF has been proposed
to underlie defi cits in hippocampal neurogenesis seen in animal models of depression. Chronic
antidepressant treatments are shown to upregulate neurotrophin expression, mediating relief
from depression. Neurotrophin-mediated proliferation of hippocampal cells may be one of the
fi nal common pathways of antidepressant effects

119
Q

Synthesis of adrenalin from noradrenalin requires which of the following
enzymes in a neurone?
A. DOPA decarboxylase
B. Dopamine hydroxylase
C. Monoamine oxidase
D. Phenylethanolamine N-methyltransferase
E. Tyrosine hydroxylase

A

D. L-tyrosine is an amino acid derived from food proteins. It is also derived from the
catabolism of phenylalanine in the liver by phenylalanine hydroxylase. L-tyrosine forms the
precursor for the catecholamine neurotransmitters. Dopamine is the major initial product
derived from L-tyrosine. Dopamine hydroxylase further converts dopamine into norepinephrine.
In cells that contain phenylethanolamine N-methyltransferase (PNMT), norepinephrine undergoes
further processing via methylation to produce epinephrine. Epinephrine is formed in trivial
amounts in the CNS but is a major product in the adrenal medulla. Any drug that enhances the
action of tyrosine hydroxylase and dopamine hydroxylase is likely to enhance noradrenergic
transmission. Catecholamines are metabolized by two enzymes: monoamine oxidase (MAO) and
catechol-O-methyl transferase (COMT).

120
Q
The majority of serotonin in the human body is found in
A. Spinal cord
B. Brain
C. Gastrointestinal tract
D. Platelets
E. Kidneys
A

C. The majority of distributed serotonin in human body is located in the intestines. Due
to the wide distribution of serotonin receptors, side-effects of serotonergic drugs may be
variable; for example 5-HT3 receptors in the area postrema or the hypothalamus are associated
with nausea and vomiting. The receptors in the basal ganglia are associated with akathisia and
agitation. Limbic receptors are associated with an anxiety response when serotonergic drugs are
administered initially. The serotonin receptors in spinal cord may produce sexual dysfunction. The
intestinal receptors constitute nearly 90% of the body’s serotonin receptors. Hence the common
side-effects with most serotonergic drugs are gastrointestinal upset and diarrhoea.

121
Q

Which of the following is a feature of a second messenger?
A. They are a class of neurotransmitters.
B. They are local hormones secreted by neurones into the blood stream.
C. They are restricted to the peripheral nervous system.
D. They combine with neurotransmitters in the nucleus.
E. They mediate the intracellular response to a neurotransmitter.

A

E. The neurotransmitter that brings a signal to a neurone is considered to be the ‘fi rst
messenger’. For the signal to get across to the postsynaptic neurone, it must be transformed into
an intraneuronal message. This is enabled via formation of the ‘second-messenger’ molecules.
Second messengers generally do not act outside the cell of origin. The most commonly
encountered second messengers include cAMP and cGMP, the calcium ion (Ca2+), and the
phosphoinositol metabolites such as inositol triphosphate (IP3) and diacylglycerol (DAG). Gases
such as nitric oxide and carbon monoxide also act as intraneuronal second-messenger molecules.
The second messengers are not hormones as they do not reach tissues via the blood stream.
Unlike receptor proteins, they do not combine with the neurotransmitter molecules directly.
They are present throughout the CNS.

122
Q

A 55-year-old man presents to A&E dreading that he has had a stroke. He
has a weakness on the right side of his face with drooling of saliva from the
right corner of his mouth. On examination he is not able to close his right
eye fully and cannot hold air against his right cheek. When attempting
a wrinkle, the right eyebrow appears sluggish. He is not able to whistle
properly. He has normal tone and power in all four limbs. Which of the
following clinical signs can be expected?
A. Bell’s phenomenon
B. Nystagmus
C. Miosis of left eye
D. Plantar extensor
E. Mydriasis of right eye

A

A. This patient’s presentation is suggestive of Bell’s palsy. It is the most common cause
of facial paralysis, usually occurring on one side only. The lifetime prevalence is about 1 in 60
in the UK. It is most commonly seen between the ages of 15 and 45, in both men and women.
Pregnancy and diabetes may increase the risk of Bell’s palsy substantially. Though the exact
cause is not known, a viral aetiology is suspected (herpesvirus). The symptoms usually develop
overnight. Most patients present with diffi culty closing the eye, drooling of saliva, and sagging of
the eyebrow on one side. Less commonly, patients may have heightened sensitivity to loud noise
on the affected side. Most patients (nearly 80%) recover completely within 3 weeks. Almost
all patients recover within 6 months. Patients with Bell’s palsy exhibit Bell’s phenomenon. Bell’s
phenomenon is a normal defence refl ex present in about 75% of the population. It results in
elevation of the globes when shutting eyes closed or when the eyes are directly threatened by
external agents. Such upward movement helps to protect the most important structures (cornea
and lens) of one’s eyes. This elevation becomes noticeable when the orbicularis muscle becomes
weak as in Bell’s palsy. Bilateral Bell’s phenomenon is found in myasthenia gravis, sarcoidosis,
bilateral Bell’s palsies, congenital facial diplegia, some rare forms of muscular dystrophy, motor
neurone disease, and Guillain–Barré syndrome.

123
Q

Regarding the serotonin (5-HT) system in the brain, which of the follow
statements is false?
A. Serotonergic cells are localized in the raphe nuclei.
B. Serotonergic cells project to virtually all areas of the brain.
C. 5-HT receptors are mostly ionotropic.
D. Serotonin does not cross the blood–brain barrier.
E. Serotonin is synthesized from tryptophan

A

C. Serotonergic cells are localized in the brainstem in a group of nuclei called the raphe
nuclei. In contrast to the more circumscribed dopaminergic pathways, almost all parts of
the brain receive serotonergic input. This probably explains the multiple effects of serotonin
receptors on mood and behaviour. All serotonin receptors are G-protein linked, except
5-HT3 which is ligand gated. Serotonin does not cross the blood–brain barrier and thus the
brain synthesizes its own serotonin. This is in turn determined by concentrations of free
plasma tryptophan and transport across the blood–brain barrier. This forms the basis of the
rapid tryptophan depletion test. Tryptophan hydroxylase hydroxylates tryptophan to 5-hydroxy
tryptophan (5-HTP), which is further decarboxylated to serotonin (5-HT) by aromatic-L-amino
acid decarboxylase (AADC) in the presence of vitamin B6 as coenzyme. The rate-limiting step
in serotonin synthesis is considered to be the availability of 5-HTP. In parallel with dopamine
and norepinephrine, following release into the synaptic cleft, 5-HT is either metabolized or
actively transported back into the neurone by a high-affi nity transporter, the serotonin reuptake
transporter (SERT). SERT is encoded by a single gene on chromosome 17. The SERT gene has
been recently postulated to play an important role in gene–environment interaction in disorders
such as depression.

124
Q

Prion protein PrPC is seen in normal cells. Pathological changes in this
protein can lead to neurodegenerative changes seen in Cruetzfeldt–
Jakob disease. Which of the following explanations in most likely for the
pathological variation?
A. The pathological protein has a different amino-acid sequence compared to the normal
protein.
B. The pathological protein differs from normal protein in the quantity produced but not
in the quality.
C. The pathological protein is structurally different from normal protein.
D. The pathological protein is coded by a genetically transmitted mutant DNA.
E. The pathological protein suppresses the immune system.

A

C. Prion protein (PrP) is a glycoprotein anchored to neuronal cell membranes. The normal
function of prion protein is not known. Bovine spongiform encephalopathy (‘mad cow disease’)
and Crutzfeldt–Jakob disease are associated with altered prion proteins. In prion diseases, the
normal cellular form of PrP (called PrPC) undergoes transformation to an altered version (called
scrapie-associated prion protein PrPSc). The latter accumulates in the brain to form insoluble
aggregates, leading to neuronal dysfunction, but unlike other neurodegenerative diseases,
prion diseases are transmissible. This is made possible because PrPSc imprints its pathological
conformation onto other, normal PrPC molecules, thus ‘converting’ them to be abnormal. PrPC
and PrPSc do not differ in their amino acid sequences

125
Q
Which of the following brain regions is involved in the regulation of arousal
and sleep–wake cycle?
A. Amygdala
B. Cingulate cortex
C. Hippocampus
D. Reticular system
E. Ventral striatum
A

D. The establishment and maintenance of a wakeful state is called arousal. In humans,
arousal activity requires at least three brain regions. The most important of these is the ascending
reticular activating system (ARAS). ARAS may have a role in setting the level of consciousness.
ARAS, via the intralaminar nuclei of the thalamus, project widely throughout the cerebral cortex.
A synchronized, rhythmic burst of neuronal activity (20–40 Hz frequency) results from ARAS and
thalamic coordination. The degree of synchronization varies directly with the level of wakefulness.
During sleep the synchronicity is lost. The third most important region in arousal state is the right
frontal lobe. The right frontal lobe is essential for the ‘maintenance’ of attention; this is evident
when testing letter-cancellation tasks or trail-making tasks in patients with right frontal lesions.
Sadock BJ

126
Q
The resting membrane potential of a neurone is estimated to be
approximately
A. +70 mV
B. +90 mV
C. 0 mV
D. −30 V
E. −70 mV
A

E. Hodgkin and Huxley showed that the inside of a cell, such as a neurone, is negatively
charged compared to the outside. This is called the resting membrane potential and its value
ranges between −40 mV and −90 mV (average −70 mV), depending on the type of cell. This
negative resting membrane potential arises due to the membranes of the resting neurone
being more permeable to potassium ions than to any other ions. There are more potassium
ions inside the cell than outside it, which is due to a constant outward leak creating a negative
potential inside the cell. An action potential is initiated in the axon hillock when the synaptic
signals received by the dendrites and soma are suffi cient to raise the intracellular potential from
−70 mV to the threshold potential of −55 mV. When this potential is reached, the Na+ channels,
which are dormant at rest, will open. This Na+ infl ux causes rapid reversal of the membrane
potential from −70 to +40 mV.

127
Q

Which of the following statements with respect to the neurotransmitter
glycine is correct?
A. It is primarily an excitatory neurotransmitter.
B. It acts via GABAA receptors.
C. It is associated with idiopathic epilepsy.
D. It facilitates chloride ion entry into neurones.
E. It is most abundant in dorsolateral prefrontal cortex

A

D. Glycine is an inhibitory transmitter, predominantly found in the spinal cord; it has
relatively insignifi cant effects in the brain compared to the spinal cord. Glycine acts on a chloride
channel that is different from GABAA receptors, called the strychnine-sensitive glycine receptor.
On activation, the transmembrane glycine receptors facilitate entry of chloride ion into the cell,
leading to hyperpolarization of the cell. Glycine is synthesized from serine; this conversion is
mediated by rate-limiting steps via serine trans-hydroxymethylase and glycerate dehydrogenase.
Glycine also serves as an obligatory neurotransmitter adjunct for glutamate activity on the
NMDA glutamate receptors (non-strychnine-sensitive glycine site). Some clinical trials have
shown a reduction in the negative symptoms of schizophrenia using D-serine or glycine mediated
via NMDA receptors, though this has not been widely replicated

128
Q

Direct measurement of cerebral glucose metabolism is possible using
which of the following methods?
A. Single photon emission computed tomography
B. Computed tomography with radiocontrast
C. Positron emission tomography
D. Functional magnetic resonance imaging
E. Magnetoencephalography

A

C. Positron emission tomography (PET) is considered as the gold standard of functional
neuroimaging modalities. Among other available techniques, only PET can measure cerebral
glucose metabolism directly. In addition, a large number of radioligands for receptor
characterization are available for PET. Despite these advantages, PET is not widely available due
to the expensive nature of the equipment; it requires relatively rapid access to a cyclotron to
produce the positron-emitting radionuclides.

129
Q
A nocturnal surge in the level of growth hormones is observed during
which of the following stages of sleep?
A. REM sleep
B. Stage 1 and 2 NREM sleep
C. Stage 3 and 4 NREM sleep
D. All stages of sleep
E. Upon awakening
A

C. Most hormones show a diurnal–nocturnal variation in their plasma levels. This is partly
related to circadian mechanisms and hypothalamic functions. Level of physical activity and diurnal
change in metabolic requirements may also infl uence the hormonal levels in blood. Growth
hormone regulates carbohydrate and lipid metabolism. There is a nocturnal surge of growth
hormone seen during slow wave sleep (stage 3 and 4 NREM sleep). Speculative association of
growth hormone surge and memory consolidation has not been borne out by experimental
results.

130
Q

A 15-year-old boy is brought to paediatric A&E by his parents. He
complains of severe headache and vomits while waiting to be seen. He
gives a history of polyuria and confusion with visual fi eld defects. On
examination he appears to be signifi cantly shorter than average height for
his age. He does not have signs of meningitis. His past medical history and
family history reveals no additional clues to his presentation. He denies
using drugs. Which of the following investigations may help in clinching the
diagnosis?
A. CSF analysis
B. Electroencephalogram
C. CT brain scan
D. Neuropsychological testing
E. Urine drug screen

A

C. The presentation here is consistent with craniopharyngioma. It is a benign, slowgrowing
tumour involving the pituitary gland. The clinical presentation is usually insidious with
the most common presenting symptoms being headache, endocrine dysfunction, and visual
fi eld disturbances. The headache may be positional and slowly progressive. Hypothyroidism,
adrenal failure, and diabetes insipidus are the common endocrine disturbances noted. Young
patients may have growth failure and/or delayed puberty. Visual fi eld defects are due to pressure
effects on the optic nerve route. Bitemporal hemianopia is the most common problem due to
pressure at the chiasma. The anatomic location of the craniopharyngioma may be classifi ed into
prechiasmal (associated with optic atrophy), retrochiasmal (associated with signs of increased
intracranial pressure such as papilledema), or intrasellar (associated with headache and endocrine
dysfunction). The diagnosis is strongly suggested by imaging studies. The appearance of a
suprasellar or intrasellar calcifi ed cyst is considered to be the radiological hallmark. CT scan is
the most sensitive method as calcifi cations are easily picked up as high-density areas. MRI is very
helpful for neurosurgeons to plan a surgical approach.

131
Q
Which of the following structures may be causally related to the
presentation in Question 131?
A. Alar plate
B. C cells of thyroid
C. Isthumus of thyroid
D. Neural tube
E. Rathke’s pouch
A

E. The embryogenetic theory of craniopharyngioma suggests the involvement of Rathke’s
pouch. The infundibulum is a downward projection of diencephalon; the Rathke’s pouch is an
upward elongation of the primitive oral cavity. Rathke’s pouch and the infundibulum develop
during the fourth week of gestation and grow towards each other until they unite to form the
hypophysis of the pituitary gland during the second month. The remnants of Rathke’s pouch
involute into a cleft and disappear completely in normal conditions. In some cases, this Rathke’s
cleft remnant can become the site of origin of craniopharyngiomas. Alar plate is related to
development of the spinal cord. Isthumus of thyroid is related to thyroglossal duct.

132
Q

A 45-year-old man presents with sudden onset, uncontrolled, wide fl inging
movements of his left arm and leg. He is a known diabetic and has not
been taking his antihypertensive medications for the last 2 months. He is
cognitively intact. Which of the following is the most likely site of a lesion?
A. Left cerebellum
B. Left putamen
C. Right internal capsule
D. Right subthalamic nucleus
E. Right thalamus

A

D. The patient described in Question 133 has an acute onset of fl inging movements of one
side of the body. This is called hemiballism. Movements are usually involuntary, wide amplitude,
and irregular with no pattern or rhythm. They commonly involve the arm and leg together; facial
involvement has been reported in about half of the cases. Movements are increased with action,
decreased with relaxation, and absent during sleep. In some patients the movements can cause
physical exhaustion or injury of the affected limb. Hemiballism is considered to be primarily a
disorder of the contralateral subthalamic nuclei (STN), but recently lesions outside STN have
been demonstrated in patients with hemiballism. It is noted that the hemiballism caused by
lesions of the STN is more severe than that caused by lesions elsewhere.

133
Q

Phineas Gage is a celebrated case in neuropsychiatry. This 25-year-old
railroad foreman sustained an extraordinary brain injury after which he
had a signifi cant change in his personality characterized by childishness,
stubborn, and obstinate behaviour with frequent use of profanities. Which
of the following brain regions was damaged signifi cantly to result in this
presentation?
A. Inferior parietal cortex
B. Superior temporal cortex
C. Orbitofrontal cortex
D. Hippocampus
E. Hypothalamus

A

C. The cardinal function of prefrontal cortex is planning and organization of behaviour
(executive function) in addition to functions of short-term memory, motor attention, and
inhibitory control. The change in Gage’s personality would be consistent with damage to the
orbitofrontal cortex of the ventral aspect of his frontal lobe, affecting affect and emotion.
A pseudopsychopathic syndrome, characterized by impulsivity and socially inappropriate
behaviour, is recognized in orbitofrontal damage. Inferior parietal cortex may play a role in
attention and body image function. The primary function of superior temporal cortex is auditory
processing. The hippocampus is the seat of episodic memory; lesions can result in amnesia. The
hypothalamus is involved in appetitive behaviours such as hunger, thirst, and sex.

134
Q

A 60-year-old man presents with a 4-week history of right foot pain
and sensory loss. The GP prescribes an analgesic and makes a referral
to a neurologist. While awaiting the specialist appointment, the patient
develops left-sided wrist drop and weakness of right hand grip. He does
not have any autonomic disturbances and appears to be cognitively intact.
The most common cause of the above presentation is
A. Diabetes mellitus
B. Periarteritis nodosa
C. Sarcoidosis
D. Leprosy
E. Temporal arteritis

A

A. Mononeuritis multiplex is a form of peripheral neuropathy wherein axonal destruction
occurs in different sites leading to sensory and motor defi cits. The nerves involved are generally
multiple and random with no predictability of progression. This acute or subacute involvement
of multiple individual nerves may be serial or even simultaneous. It is not a single disease but
a syndrome caused by various disorders; often the fi nal common pathway includes vascular
damage to neurones. Nearly one-third of cases are idiopathic. The most common identifi able
cause in adults is diabetes mellitus, followed by vasculitis syndromes such as polyarteritis nodosa
and connective tissue diseases such as rheumatoid arthritis or systemic lupus erythematosus. In
children and adolescents, the most common cause of mononeuritis multiplex is connective tissue
disease. Mononeuritis multiplex can mimic conversion disorder and present to psychiatric units.

135
Q

A neuropsychologist administers a test to a 21-year-old man with a change
in personality related to brain damage. The patient is asked to name
the colour of a word while ignoring the actual word. What test is being
administered?
A. Wisconsin Card Sorting Test
B. Benton Visuomotor Test
C. Rorschach’s Test
D. Continuous Performance Test
E. Stroop Test

A

E. The question describes the classic form of the Stroop Colour–Word Test. In this test
the subject is initially required to read names of some basic colours. Later the subject is asked
to name the colours of geometrical shapes. Following this, the test of interference is applied.
Looking at a colour name written in a different colour produces a confl ict; this makes the subject
read the name instead of saying the colour in which it is written, for example if the word ‘blue’
is written in green the subject tends to say blue, even when asked to name the displayed colour.
The classic form of the Stroop Colour–Word Test is the most commonly used, though variations
such as Emotional Stroop are now available. Rorschach’s is a projective test which uses ink-blot
images. The Continuous Performance Test measures sustained/selective attention and impulsivity.
Goldfarb L

136
Q
Which of the following is the rate-limiting enzyme in the synthesis of
dopamine?
A. Tyrosine hydroxylase
B. DOPA decarboxylase
C. Dopamine hydroxylase
D. Monoamine oxydase
E. Catechol-O-methyl transferase
A

A. Dopamine is synthesized from the amino acid tyrosine. Initially, tyrosine is converted
to L-dihydroxyphenylalanine (L-DOPA) by tyrosine hydroxylase (the rate-limiting step). L-DOPA
is rapidly converted to dopamine by dopa decarboxylase. Dopamine is stored in vesicles
and 80% of the released dopamine is rapidly transported back into the nerve terminal by a
dopamine-specifi c transporter (DAT). This intracellular extravesicular dopamine is metabolized
by monoamine oxidase (MAO) to dihydroxyphenylacetic acid (DOPAC). Twenty percent of the
released dopamine is sequentially degraded extracellularly by catechol-O-methyltransferase
(COMT) and MAO to 3-methoxytyramine (3-MT) and homovanillic acid (HVA). Dopaminergic
cell bodies in the brain are mainly localized in the ventral tegmental area in the brainstem.
There are predominantly four pathways that are considered to be dopaminergic in the brain:
mesocortical and mesolimbic axons originate from the VTA and project to the prefrontal cortex
and limbic structures, respectively; the tuberoinfundibular pathway mediates release of prolactin
from the pituitary; and the nigrostriate pathway forms an integral part of the basal ganglion
extrapyramidal system.

137
Q

A 55-year-old patient presents with bilateral hand tremors that worsen
with stress. Which of the following features of the tremor is suggestive of
benign essential tremor rather than Parkinsonism?
A. Tremor is increased by alcohol
B. Tremor is reduced by action
C. Tremor is absent at rest
D. Frequency of the tremor is 4–6 cycles per second
E. The amplitude of the tremor is large

A

C. Parkinsonian tremor is usually of large amplitude with a frequency of 4 to 6 cycles
per second. It is a resting tremor which persists even during action. Parkinsonian tremor is not
reduced by alcohol but is exaggerated in stressful situations. In contrast, benign essential tremor
is usually of smaller amplitude and higher frequency (10 to 12 cycles per second). It is often
seen during action and may be unnoticeable during rest. It is exacerbated by stress, similar to
Parkinsonian tremor. Clinically, essential tremor is similar to exaggerated physiological tremor.

138
Q

A 50-year-old woman, being treated for depression as an outpatient,
presents to A&E with acute-onset, severe headache. She describes
this as the ‘worst headache’ she has ever had in her life. She insists on
switching off the lights in the examination cubicle. Her blood pressure is
150/90 mmHg. When asked to get up from the examination couch, she
complains of neck stiffness. An emergency CT scan is normal. The next
most appropriate step is
A. Change her antidepressant
B. Obtain MRI scan
C. Obtain urine drug screen
D. Prepare for lumbar puncture
E. Prescribe haloperidol 2.5 mg only

A

D. The patient described here has acute, severe headache, photophobia, and meningism.
These features are highly suggestive of subarachnoid haemorrhage (SAH). A CT scan is not
100% sensitive in ruling out possible intracranial bleed. Given the high clinical suspicion, the
gold standard test for SAH, lumbar puncture, must be carried out. Presence of depression
must not distract one from considering acute medical causes of somatic complaints such as
headaches. Haloperidol is not indicated in this scenario. Please note that if the CT scan discloses
a subarachnoid haemorrhage, lumbar puncture need not be carried out as a routine

139
Q

A 68-year-old woman has long-standing hypertension. She is diagnosed
to have somatization disorder by her GP and is prescribed venlafaxine
225 mg/day. Unfortunately she develops a cerebrovascular accident.
While being treated for stroke at the acute neurology unit, she starts
having severe, ‘gruesome’ pain on her left side of the body. The pain has an
intense, scalding quality. The most likely site of infarct is
A. Cerebellum
B. Cerebello pontine junction
C. Thalamus
D. Hippocampus
E. Internal capsule

A

C. This patient is having an infarct of the thalamus. Thalamic infarcts affecting ventral
posterolateral nucleus and posteromedial nucleus result in a severe sensory syndrome
characterized by intense burning pain, hyperaesthesia, or hemianaesthesia affecting the
contralateral body. Cold thermal stimuli, emotional stress, and loud sounds may aggravate
the painful state. Despite this apparent hypersensitivity, the patient shows an elevated pain
threshold requiring a stronger than usual stimulus to produce a sensation of pain (hypoalgesia
with hyperpathia). This thalamic pain syndrome is also called Dejerine–Roussy syndrome. Some
patients may develop hemiataxia and choreoathetosis

140
Q

Brain region associated:

anomia and dysgraphia

A

parietal cortex

141
Q

Brain region associated:

nystagmus and ataxia

A

cerebellum

142
Q

Brain region associated:

word blindness and movement agnosia

A

occipital cortex

143
Q

Brain region associated:

prosopagnosia and fluent aphasia

A

temporal cortex

144
Q

Brain region associated:

dysphagia and sleep apnea

A

brain stem

145
Q

3 areas lacking BBB

A

area postrema medulla
subfornical organ
neurohypophysis

146
Q

Organisation of mature brain

A

Primary->secondary->contents

  1. Rhobencephalon->myelencephalon & metencephalon-> caudal medulla oblongata & pons, part of medulla and cerebellum
  2. Mesencephalon->mesencephalon->tectum (inf sup colliculi), basis pedunculi, tegmentum (red nucleus, fibre tracts)
  3. Prosencephalon->diencephalon (to thalamus, epithalamus hypothalamus, subthalamus) & telencephalon (to hemispheres, olfactor, corpus striatum, medullary centre, pallium
147
Q

Contents of medulla

A
  1. pyramid - consists of corticospinal fibres (a.k.a. pyramidal tract)
  2. the olive - marks the position of the inferior olivary nucleus
  3. fasciculi gracilis and cuneatus
  4. cranial nerves:
    a) abducens (VI)
    b) facial (VII)
    c) vestibulocochlear (VIII)
    d) glossopharyngeal (IX)
    e) vagus (X)
    f) accessory (XI)
    g) hypoglossal (XII)
148
Q

Contents of pons

A

The Pons
· consists of a basal portion (a relay station between the cortex of a cerebral
hemisphere and the opposite cerebellar hemisphere - involved in voluntary
movement) and a dorsal portion (contains ascending and descending tracts)
· contents:
1. trigeminal (V) nerve nucleus
2. motor nucleus of facial (VII) nerve
3. vestibular (VIII) nuclei
4. abducens (VI) nerve nucleus
5. isthmus of brain stem - contains the lateral lemniscus which is the main
auditory tract of the brain stem
6. reticular formation
7. locus coeruleus – lies at the rostral end of the fourth ventricle
8. trapezoid body - contains acoustic fibres
9. medial longitudinal fasciculus

149
Q

Contents of midbrain

A

Midbrain
· extends from the pons to the mammillary bodies of the diencephalon
· contents:
1. inferior and superior colliculi
a) the inferior colliculus is a relay nucleus on the auditory pathway to the
thalamus and thence to the cerebral cortex
b) the superior colliculus is involved in the voluntary control of ocular
movements, and in movements of the eyes and head in response to
visual and other stimuli
2. oculomotor (III) nerve
3. trochlear (IV) nerve
4. raphe nuclei
5. red nucleus
a) motor nucleus; has afferents from cerebellum and cerebral cortex
6. substantia nigra
7. ventral tegmental area
8. crus cerebri

150
Q

Pathways of SN

A
Afferent pathways:
· corpus striatum
· subthalamic nucleus
· raphe nuclei of midbrain
· pontine reticular formation
Efferent pathways:
· neostriatum (caudate nucleus)
· amygdaloid body in the temporal lobe
151
Q

Medial medullary syndrome

A

Medial medullary syndrome
· results from occlusion of the medullary branch of the vertebral artery
· results in:
· contralateral hemiparesis
· contralateral disturbance of the sensations of position and movement
· paralysis of the tongue is ipsilateral, and deviation is towards the affected
side

152
Q

Lateral medullary syndrome

A

Lateral medullary (Wallenberg’s) syndrome
· due to occlusion of a vessel supplying the lateral area of the medulla
· results in:
· ipsilateral loss of pain and temperature in the area of distribution of the
trigeminal nerve
· contralateral sensory loss - touch sensation is diminished rather than
abolished (since the medial lemniscus is intact)

153
Q

Tracts of raphe nucelus

A

Raphe nuclei
· the neurones of the raphe produce serotonin and use this as a neurotransmitter
Afferents:
· spinal cord
· primary motor cortex of the frontal lobe
· parietal lobe
· vestibular nuclei
· cerebellar nuclei
· periaqueductal grey matter
Efferents:
· thalamus
· spinal cord
· activity of the raphe nuclei induces sleep, due to release of serotonin in more rostral
parts of the brain
· electrical stimulation of the periaqueductal grey matter or the nucleus raphe magnus
results in loss of pain sensation
· encephalins are released at synapses in the periaqueductal grey matter, the raphe
nuclei, and the substantia gelatinosa in the spinal cord - the analgesic effects of these
chemicals requires integrity of the raphe spinal tract

154
Q

Reticular activation system mechanism for consciousness

A

The Reticular Activating System (RAS)
· sensory data conveyed by the lemniscal system (medial, spinal, and trigeminal
lemnisci) when projected from the thalamus to the somaesthetic cortical area, is
interpreted in a specific manner with respect to the nature of the stimulus
· much of the cortex is stimulated, with an effect on levels of consciousness and on
alerting reactions to sensory stimuli
· impulses from the trigeminal area of distribution have a significant effect on
consciousness
· some general anaesthetics are thought to suppress transmission through the RAS,
although conduction continues along lemniscal routes
· prolonged coma results from damage to the pontine or mesencephalic reticular
formation

155
Q

Networks for LC

A
The Locus coeruleus
· cells contain large quantities of noradrenaline
· Afferents:
· some raphe nuclei
· hypothalamus
· amygdala and cingulate gyrus
· Efferents:
· cerebral cortex
· diencephalon
· brain stem
· cerebellum
· spinal cord
156
Q

Structures of diencephalon

A

Structure
· consists of 4 components on each side:
1. thalamus:
a) subdivided into nuclei which have different afferent and efferent
connections
2. subthalamus:
a) consists of the subthalamic nucleus, and also parts of the reticular
formation and substantia nigra
3. epithalamus:
a) includes the pineal gland, and structures concerned with autonomic
responses to emotional changes
4. hypothalamus:
a) the main cerebral centre for integrative control of the autonomic
nervous system and of several endocrine glands
b) the neurohypophysis which includes part of the pituitary gland is an
outgrowth of the hypothalamus

157
Q

Structure/nuclei of the thalamus

A

Thalamus
· there are a number of thalamic nuclei, all of which except the reticular nucleus sends
axons to the cerebral cortex
· there are few if any connections between the nuclei of the thalamus

Intralaminar Nuclei

  1. centromedian nucleus
  2. parafascicular nucleus

Ventral Group of Nuclei
1. Medial geniculate body
a) involved in discriminative aspects of hearing
2. Lateral geniculate body/ nucleus (LGN)
a) contains a detailed point-to-point projection of the retina
b) involved in awareness of visual stimuli and the discriminative and
mnemonic aspects of vision

  1. Ventral Posterior Nucleus
    a) part of the pathway for conscious appreciation of sensations arising
    from skin, muscles, and internal parts of the body
    b) contains a topographical projection of the contralateral half of the body
  2. Ventral Lateral Nucleus
  3. Ventral Anterior Nucleus
  4. Ventral Medial Nuclei

Posterior group of nuclei

  1. part of the pulvinar
  2. part of the medial geniculate body
  3. suprageniculate nucleus
  4. nucleus limitans

Lateral group of nuclei

  1. Lateral Dorsal Nucleus
    a) part of the limbic system
  2. Lateral Posterior Nucleus
    a) projects to the somatosensory association cortex of the parietal lobe
  3. Pulvinar

Medial group of nuclei
1. Mediodorsal Nucleus
a) part of the limbic system and contributes to those aspects of emotions
considered as ‘moods’
b) lesions around the third ventricle involving the mediodorsal nucleus are
seen in Korsakoff’s syndrome and this area probably has a role in
memory
c) memory deficits have been reported following surgical destruction of
the mediodorsal thalamic nuclei
2. Medioventral Nucleus
a) part of the limbic system

Anterior group of nuclei
1. consists of three nuclei which are considered to be part of the limbic system

158
Q

Subthalamus structure

A

Subthalamus

  1. sensory fasciculi
  2. rostral extensions of midbrain nuclei
  3. fibre bundles from the cerebellum and globus pallidus
  4. subthalamic nucleus
    a) a motor nucleus
    b) a lesion in this nucleus results in hemiballismus
159
Q

Hypothalamus functions

A

Hypothalamus
· surrounds the third ventricle
· functions:
· autonomic control of basic body functions, e.g., heart, bowel, blood
pressure
· regulation of body temperature
· regulation of food and water intake
· experience of pleasure
· rage and aversion
· the amygdala and hypothalamus have sex-specific patterns of neuronal and dendritic
development
· the anterior hypothalamus is involved in the relaxation response
· the posterior hypothalamus is involved in the fight-or-fright response

160
Q

Sham mirth and sham rage-> hypothalamus damage

A

uncontrolled laughter (sham mirth) occurs in hypophyseal and hypothalamic
tumours
· hypothalamic rage (sham rage) due to stimulation of hypothalamic nucleus

161
Q

Hypothalamic nuclei

A
  1. Supraoptic nucleus
  2. Paraventricular nucleus
    a) involved in the eating of certain types of food
  3. Suprachiasmatic nucleus
    a) forms the brain’s timepiece; keeps an approximate 24 hr clock
  4. Ventromedial nucleus
    a) satiety centre which inhibits food intake
    b) controls parasympathetic activities
  5. Dorsomedial nucleus
  6. Infundibular nucleus
  7. Premammillary nucleus
  8. Mammillary body and posterior nucleus
  9. Lateral nucleus
    a) involved in the stimulation of eating and control of drinking
    b) controls sympathetic activities, and is involved in the somatic correlates
    of heightened emotionality
162
Q

Efferents of hypothalamus

A

Efferent pathways

  1. periventricular fibres
  2. dorsal longitudinal fasciculus
  3. mamillothalamic fasciculus
163
Q

Amygdala afferents, efferents

A
Amygdala
· ‘the seat of social and emotional intelligence’
· consists of two main nuclei - medial and lateral
Medial amygdala
· has afferents from olfactory tract
· rich in encephalin-containing cells
Lateral amygdala
· able to modify output from hypothalamus and initiate appropriate higher functions,
i.e. hypothalamus signals hunger and the amygdala signals the correct response from
the body
Afferents
· the contralateral amygdala
· dopaminergic brain stem nuclei
· frontal association area
· lateral olfactory stria
· noradrenergic brain stem nuclei
· serotonergic brain stem nuclei
· septal nucleus
· temporal association area
Efferents
· hypothalamus
· septal nucleus
· corpus striatum
· frontal association area
· lateral olfactory stria
· temporal association area
· thalamus
164
Q

Functions of hippocampus

A

Functions
· main role is that of attention, information processing, memory, new learning,
cognitive mapping of the environment
· prevents extremes of arousal by maintaining a state of quiet alertness
· most complex organically-produced hallucinations are caused by tumours of the
most anterior part of the temporal lobe (including the amygdala and hippocampus)
· left sided lesions affect verbal memory
· right sided lesions affect visual memory

165
Q

Role of anterior cingulate gyrus

A

associated with processing and modulating the expression of emotional nuances,
emotional learning and vocalization, formation of long term attachments, and
maternal behaviour

166
Q

Who identified the limbic system as responsible for the cortical representations of emotions

A

The first evidence that the limbic system was responsible for the cortical representation of emotions was discovered in 1939, by Heinrich Kluver and Paul Bucy. Kluver and Bucy, after much research, demonstrated that the bilateral removal of the temporal lobes in monkeys created an extreme behavioral syndrome.

167
Q

Limbic lobe identified by who

A

Paul Pierre Broca in 1878 spoke of ‘le grand lobe limbique’ or the great limbic lobe and applied the term “limbic” (from the Latin limbus for border) to the curved rim of the cortex which incudes the cingulate and the parahippocampal gyri.

168
Q

Primary nuclei of limbic system

A

Primary nuclei

  1. hypothalamic nuclei
  2. amygdaloid nucleus
  3. hippocampus
  4. septal nuclei
  5. thalamic nuclei:
    a) anterior, lateral dorsal, and medioventral nuclei
  6. mamilllary bodies
  7. superior central nucleus
  8. ventral tegmental area
  9. raphe nucleus
169
Q

Cortical areas of limbic system

A

Cortical areas

  1. cingulate gyrus
  2. hippocampal formation:
    a) dendate nucleus
    b) hippocampus
    c) parahippocampal gyrus
  3. olfactory tubercle
  4. secondary olfactory area
  5. subcallosal gyrus
  6. indusium griseum
  7. paraterminal gyrus
170
Q

Connecting tracts of limbic

A

Connecting tracts

  1. cingulum
    a) hippocampus to cingulate gyrus
  2. anterior commissure
  3. dorsal longitudinal fasciculus
  4. stria terminalis
    a) hypothalamus to amygdala
  5. stria medullaris thalami
  6. amygdalofugal bundle
    a) hypothalamus to amygdala
  7. fornix
    a) hypothalamus to hippocampus
  8. medial forebrain bundle
    a) hypothalamus to septal nuclei
    b) involved in the positive reinforcement of behaviours
  9. mamillothalamic tract
    a) mamillary bodies (hypothalamus) to anterior thalamic nucleus
171
Q

Circuits of the limbic system

A
Circuits of the limbic system
· the largest components contain a ring of interconnected neurons called the circuit
of Papez, which links the neocortex to the limbic system via the cingulate gyrus
· hippocampus
· mammillary body
· thalamus
· cingulate gyrus
· input:
· neocortex
· thalamus
· septal area
· raphe nuclei
· ventral tegmental area
· catecholamine nuclei of the reticular formation
· output:
· neocortex
· regions of the reticular formation that influence the autonomic system
indirectly
172
Q

Functional considerations with damage to limbic system

A

Functional considerations
· bilateral removal of the temporal lobes (including the hippocampal formations and
amygdaloid bodies) is followed by docility and lack of emotional responses such as
fear or anger
· there is increased sexual activity, often perverted
· lesions to the amygdala produce similar changes, although the sexual behaviour is
less affected
· electrical stimulation of the amygdala in humans induces feelings of fear or anger
· summary of functions:
· strong affective responses such as fear, anger, and sexual behaviour
· changes in visceral function:
· changes in gastrointestinal movements and secretion
· piloerection
· pupillary dilatation
· changes in respiratory movements
· role in memory (especially the hippocampus)
· the hippocampus may also have a role in the sleep-wake cycle

173
Q

Memory disturbance occurs when damage to what components of limbic

A
· memory disturbance occurs in damage to:
· mammillary bodies
· hippocampus
· fornix
· substantia innominata
174
Q

Structure of the corpus striatum

A

The Corpus Striatum
· a substantial region of grey matter near the base of each cerebral hemisphere
· the term ‘basal ganglia’ refers to the corpus striatum, subthalamic nucleus, and
substantia nigra
· consists of:
1. caudate nucleus
2. lentiform nucleus
a) putamen
b) globus pallidus
· the paleostriatum (a.k.a. pallidum) refers to the globus pallidus in mammals
· the neostriatum (a.k.a. striatum) refers to the putamen and the caudate nucleus

175
Q

Functions of the stiatum

A

Functions
· the striatum inhibits the pallidum
· the pallidum inhibits the thalamocortical neurons
· GABA is the inhibitory transmitter
· when no movements are being made, the cells of the striatum are quiet and the cells
of the pallidum are active
· shortly before a movement, the above situation is reversed
· removal of pallidal inhibition allows the ventral thalamic nuclei to be stimulated by
other afferent fibres, most of which come from the premotor area of cortex
· the nigrostriatal dopaminergic neurons are active all the time

176
Q

Cause of huntington’s chorea

A

· Huntington’s chorea:
· neuronal degeneration in the corpus striatum, most marked in the
neostriatum

177
Q

Functional anatomy of cerebellum

A

Functional anatomy
1. vestibulocerebellum
a) consists of the flocculonodular node and a region of the vermis known
as the uvula
b) receives input from the vestibular nerve and nuclei
c) influences motor neurons through the vestibulospinal tract, the medial
longitudinal fasciculus, and reticulospinal fibres
d) involved in adjustment of muscle tone and maintenance of equilibrium
2. spinocerebellum
a) consists of parts of the vermis and adjacent parts of the hemispheres
b) site of termination of the spinocerebellar tracts, which convey
proprioceptive and other sensory information
c) receives information from all three sensory nuclei of the trigeminal
nerve, and also the red nucleus
d) involved in the control of muscle tonus and synergy of collaborating
muscles
3. pontocerebellum
a) the lateral parts of the hemispheres and the superior vermis in the
posterior lobe; also the dentate nucleus
b) fibres pass through the red nucleus
c) ensures a smooth and orderly sequence of muscular contractions and
the intended precision in the force, direction, and extent of volitional
movements
d) connections are ipsilateral

178
Q

Cells of the cerebellum

A
Cells of the cerebellum
· glomerulus:
· mossy fibre
· dendrites of granule cells
· axon of Golgi cell
· Purkinje cells:
· use GABA as neurotransmitter
· axons are the only cells leaving the cortex
· terminate in central nuclei of cerebellum
· also send efferents to:
· thalamus
· Golgi cells
· reticular neurones
· intracortical circuits are inhibitory
· most input to the cortex is excitatory
179
Q

Components of EPS

A
The Extrapyramidal System
· includes:
· basal ganglia (caudate and lentiform nucleus)
· cerebellum:
· dentate nucleus
· brain stem nuclei:
· substantia nigra
· midbrain nuclei
· tectal nuclei
· lateral vestibular nucleus (Reiter’s nucleus)
180
Q

The pyramidal system

A
The Pyramidal System
· originates in the motor cortex
· descends through:
· corona radiata
· internal capsule
· ventral midbrain
· medulla oblongata (forms the pyramids)
· 85 % of the fibres decussate in the medulla to form the lateral corticospinal tract
· 15 % of the fibres form the ventral corticospinal tract
· sends fibres to:
· red nucleus
· reticular formation
· pontine nuclei
· terminates in:
· interneurones of dorsal horn (laminae IV, V, and VI)
· ventral horn (lamina VII) – contains Renshaw cells (interneurones)
181
Q

Degeneration of pyramidal system leads to

A
Clinical considerations
· degeneration leads to:
1. ataxia
2. wide-based gait
3. loss of vibration sense
4. loss of deep pain sensation
5. positive Romberg’s sign
182
Q

Components of frontal lobes

A
The Frontal Lobes
Frontal Operculum
· consists of areas 44, 45, and 47
· Broca’s area – on the dominant side, and consists of areas 44 and 45
· a lesion in this area can lead to Broca’s/ Expressive nonfluent aphasia
· lesions in the non-dominant frontal operculum can lead to dysprosody
Superior mesial region
1. supplementary motor area (SMA)
2. anterior cingulate cortex
· lesions of the left or right superior mesial region can lead to akinetic mutism
Inferior mesial region
1. orbital cortex
a) functions include:
i) social behaviour
ii) personality
b) lesions of the orbital cortex (either side) can lead to a form of acquired
sociopathy
2. basal forebrain:
a) contains the following nuclei:
i) diagonal band of Broca
ii) nucleus accumbens
iii) septal nuclei
iv) substantia innominata
b) lesions of the basal forebrain (either side) can lead to amnesia (retrograde and
anterograde) and confabulation
Dorsolateral prefrontal cortex (DLPFC)
· functions include:
· problem-solving
· perceptual judgement
· memory
· programming and planning sequences of behaviour
· verbal regulation
· level of response emission
· adaptability of response pattern
· tertiary level of motor control
· lesions in this region can cause:
· abnormal cognitive executive functions
· impairment of verbal (left) or non-verbal (right) intellectual functions
· memory impairment affecting recency and frequency judgements
· poor organization
· poor planning
· poor abstraction
· disturbances in motor programming
· impaired verbal fluency (left-sided lesions)
· impaired design fluency (right-sided lesions)
183
Q

Frontal lobe syndrome

A
Frontal lobe syndrome
· personality change:
· disinhibition
· reduced social and ethical control
· sexual indiscretions
· poor judgement
· elevated mood (fatuous euphoria)
· perseveration
· contralateral spastic paresis
· ‘Utilization syndrome’
· palilalia
· impairment of attention, concentration, and initiative
· motor Jacksonian fits
· Witzelsucht (inappropriate jocularity)
· aphasia
· primary motor aphasia
· motor agraphia
· ipsilateral optic atrophy
· anosmia
· magnetism (ask pt. to draw a clock face)
· grasp reflex, and other primitive reflexes
· urinary incontinence if the lesion is lateral
184
Q

The parietal lobe structures and associated damage

A

The Parietal lobes
Temporoparietal junction
· left-sided lesions:
· receptive aphasia
· right-sided lesions:
· phonagnosia (inability to recognise familiar voices)
· amusia (inability to recognise and process music)
Inferior parietal lobule
1. angular gyrus
2. supramarginal gyrus
· left-sided lesions (affecting the arcuate fasciculus):
· conduction aphasia – patient cannot repeat what is said to them
· tactile agnosia
· dysphasia
· right-left disorientation
· finger agnosia (can be elicited by the ‘In-Between Test’)
· agraphia
· body image disorders
· sensory extinction
· astereoagnosia (inability to recognise objects by palpation)
· alexia
· dysgraphaesthesia (inability to recognise letters or numbers written on the hand)
· right-sided lesions:
· anosognosia (lack of awareness of disease, particularly of hemiplegia)
· neglect
· tactile agnosia
· asomatognosia (inability to recognize parts of the body)
· constructional dyspraxia
· Angular gyrus:
· inability to read
· inability to write

185
Q

Gertsmann’s syndrome

A
Gerstmann’s syndrome
· lesion of dominant parietal lobe:
1. right-left disorientation
2. finger agnosia
3. dysgraphia
4. dyscalculia
186
Q

The temporal lobe structures and associated deficits

A

The Temporal lobes
Superior temporal gyrus
· area 22, forms, on the left, Wernicke’s area
· lesions here can cause a receptive aphasia
Posterior Inferolateral region
1. post. portion of the middle temporal gyrus
2. post. portion of the inferior temporal gyrus
3. post. portion of the fourth temporal gyrus
· lesions in this region can lead to:
· prosopagnosia
· impaired object recognition
Anterior Inferolateral region
1. ant. portion of the middle temporal gyrus
2. ant. portion of the inferior temporal gyrus
3. ant. portion of the fourth temporal gyrus
4. temporal pole
· left-sided lesions can lead to:
· anomia
· defects in accessing the reference lexicon
· right-sided lesions may lead to:
· inability to name facial expressions
· bilateral lesions lead to:
· retrograde amnesia
The Mesial Temporal Region
1. parahippocampal gyrus (areas 27 and 28)
2. amygdala
3. entorhinal cortex
4. hippocampus
· left-sided lesions:
· anterograde amnesia affecting verbal information
· right-sided lesions:
· anterograde amnesia affecting non-verbal information
· bilateral lesions:
· verbal and non-verbal anterograde amnesia

187
Q

Kluver-Bucy

A

· Kluver-Bucy (due to bilateral damage of amygdala, uncus, and hippocampus)
· social and emotional agnosia (unable to discern the meaning or significance
of common objects) hypermetamorphosis/ hyperorality (perseverative oral exploration)
· sexual indiscretion
· loss of fear, leading to aggression
· affective blunting
· visual agnosia

188
Q

Occipital lobes components

A

The Occipital lobes
1. primary visual cortex
2. visual association cortices
· lesions of the dorsal region and adjoining parietal region can lead to partial
(unilateral) or full-blown (bilateral) Balint’s syndrome:
1. simultanagnosia (the ability to understand individual details of a scene, but
is unable to recognize the full meaning)
2. ocular apraxia or psychic gaze paralysis
3. optic ataxia
· bilateral dorsal lesions:
· astereopsis
· impaired visual motion perception
· depending on site of lesion, the following can occur:
· achromatopsia (loss of colour perception) – due to damage to the lingual gyri
· hemiachromatopsia
· acquired dyslexia
· apperceptive visual agnosia
· visual agnosia
· prosopagnosia (unable to recognize familiar faces) - can occur with bilateral
inferior occipital lesions
· disturbances of visual recognition
· complex visual hallucinations
· visual field defects
· visual disorientation

189
Q

Anton’s syndrome

A

Anton’s syndrome - denial of visual disability and confabulation of visual detail

190
Q

Basal ganglia components and connections

A
The Basal Ganglia
Components
1. corpus striatum
a) caudate nucleus
b) lentiform nucleus
i) globus pallidus
ii) putamen
2. amygdaloid nucleus
3. claustrum
Connections
1. Afferents:
a) caudate nucleus
b) cerebral cortex
c) putamen
d) substantia nigra
2. Efferents:
a) globus pallidus
b) hypothalamus
c) reticular formation
d) substantia nigra
e) subthalamus
f) thalamic nuclei
Fronto-subcortical circuits
· Alexander et al. (1986)
· connect the frontal cortex with the basal ganglia and the thalamus
· involved in:
· motor activity
· eye movements
· behaviour
· the overall structure is:
· frontal lobe cortex Ë caudate nucleus Ë globus pallidus/ substantia nigra Ë
thalamus Ë frontal lobe cortex
191
Q

Somatosensory systems- 2

A
Somatosensory systems
Spinothalamic system
· carries information from:
1. mechanoreceptors (crude touch, pressure, tickle and itch, sexual sensation)
2. thermoreceptors (warm and cold sensation)
3. pain receptors
Dorsal column system
· carries information only from mechanoreceptors:
1. fine, precisely localised touch
2. vibration
3. kinaesthetic sensation
4. fine pressure sensation
· contains thick myelinated fibres
192
Q

Features of lower motor euron lesions

A

Lower motor neuron lesions
· features include:
1. flaccid paresis or paralysis
2. weak or absent tendon reflexes
3. muscle atrophy
4. fibrillation potentials, detected on electromyography
5. sprouting at nodes of Ranvier and at motor end plates (on biopsy)

193
Q

Upper motor neuron features

A
Upper motor neuron lesions
· features include:
1. weak or absent voluntary movements
2. atrophy does not occur
3. spasticity, due to the continuous operation of the stretch reflex
4. exaggerated tendon reflexes
5. upgoing plantar reflex
6. superficial reflexes are suppressed or absent (e.g. abdominal &amp; cremasteric
reflexes)
194
Q

Light and accommodation reflex

A
The Light and Accommodation reflexes
· involves:
1. Short ciliary nerves
2. Ciliary ganglion
3. Oculomotor nerve
4. Pre-tectal area
5. Edinger-Westphal nucleus
6. Posterior commissure
7. Optic tract
8. Lateral Geniculate nucleu
195
Q

Corneal reflex

A

The corneal reflex
· involves:
1. afferent fibres are in the ophthalmic nerve
2. efferent fibres are in the facial nerve
3. spinal nucleus of the trigeminal nerve

196
Q

Alzheimer’s disease macro and histopathology

A
Alzheimer’s disease
Macroscopic neuropathology
· global brain atrophy (most marked in the frontal and temporal lobes)
· ventricular enlargement
· sulcal widening
Histopathology
· neuronal loss
· shrinking of dendritic branching
· reactive astrocytosis
· neurofibrillary tangles
· neuritic (senile) plaques
· histological changes commonly seen in the hippocampus include:
· granulovacuolar degeneration
· Hirano bodies
· neurofibrillary tangles
· neuritic (senile) plaques
Ultrastructural pathology
· neuritic plaques contain a core of amyloid
· scattered deposits of beta-amyloid have been found to localize to active microglia
197
Q

Neurochemical pathology AD

A
Neurochemical pathology
· decreased:
· Acetylcholinesterase
· Choline acetyltransferase
· GABA
· noradrenaline
198
Q

Picks disease neuropathology

A
Pick’s disease
Macroscopic
· selective asymmetrical atrophy of the anterior temporal lobes and frontal lobes
· knife-blade gyri
· ventricular enlargement
Histopathology
· Pick’s bodies
· neuronal loss
· reactive astrocytosis
· these changes are seen in:
· cerebral cortex
· basal ganglia
· locus coeruleus
· substantia nigra
Ultrastructural
· Pick’s bodies consist of:
· straight neurofilaments
· paired helical filaments
199
Q

Lewy body disease neuropathology

A
Lewy body disease
Histopathology
· Lewy bodies
· neuronal loss
· neurofibrillary tangles
· neuritic (senile) plaques
· compared to Parkinson’s disease, the density of Lewy bodies is much higher in:
· cingulate gyrus
· parahippocampal gyrus
· temporal cortex
Ultrastructural
· Lewy bodies contain:
· protein neurofilaments
· granular material
· dense core vesicles
· microtubule assembly protein
· ubiquitin
· tau protein
200
Q

Neuropathology PD

A
Parkinson’s disease
Macroscopic
· depigmentation of the substantia nigra, especially in the zona compacta
· depigmentation of the locus coeruleus
· diffuse cortical atrophy can take place
Histopathology
· neuronal loss
· reactive astrocytosis
· Lewy bodies in:
· substantia nigra
· dorsal motor nucleus of the vagus
· hypothalamus
· nucleus basalis of Meynert
· locus coeruleus
· Edinger-Westphal nucleus
· raphe nuclei
· cerebral cortex
· olfactory bulb
· presence of melanin-containing macrophages
Neurochemical
· reduced inhibitory dopaminergic action of the nigrostriatal pathway on striatal
cholinergic neurones
201
Q

Neuropathology Huntington’s

A
Huntington’s disease
Macroscopic
· small brain with reduced mass
· marked atrophy of the corpus striatum, particularly the caudate nucleus
· marked atrophy of the cerebral cortex, particularly the frontal lobe gyri
· dilatation of the lateral and third ventricles
Histopathology
· neuronal loss in the cerebral cortex, especially the frontal cortex
· neuronal loss in the corpus striatum, particularly neurones using as
neurotransmitters:
· GABA and enkephalin
· GABA and substance P
· astrocytosis in affected regions
· sparing of:
· diaphorase-positive neurones containing nitric oxide synthase
· large cholinesterase-positive neurones
Neurochemical
· Í GABA
· Í Glutamic acid decarboxylase
· Í Acetylcholine
· Í Substance P
· Í CRF
· È somatostatin
· dopamine hypersensitivity
202
Q

Dopamine hypersensitivity hypothesis for TD

A

Dopamine hypersensitivity hypothesis
Proposed mechanism
Long- term treatment ‡ chronic dopamine receptor blockade ‡ D2 receptor
hypersensitivity in the nigrostriatal pathway ‡ tardive dyskinesia
Evidence in favour
· studies of denervation-induced hypersensitivity in muscles
· animal experiments in which, following the discontinuation of antipsychotic
drugs, acute dopamine agonist challenges ‡ increased oral stereotyped behaviour
· animal experiments in which repeated antipsychotic treatment may lead to
increased D2 receptor levels
Problems with the theory
· differences in the chronology of onset of symptoms in animal and human models
· PM studies in humans have not shown significant differences in schizophrenic
brains with or without TD
· blood biochemical assays have not shown significant differences between patients
with TD and patients without TD with respect to:
· prolactin
· somatotrophin
· dopamine agonists do not strikingly exacerbate tardive dyskinesia
· dopamine antagonist antipsychotics may sometimes worsen TD

203
Q

Theories of TD

A

Dopamine hypersensitivity
Free radical induced neurotoxicity
GABA insufficiency
Noradrenergic dysfunction

204
Q

Free radical theory of TD

A

Proposed mechanism
Long- term treatment ‡ increased catecholamine turnover ‡ free radical byproducts
‡ membrane lipid peroxidation in the basal ganglia ‡ tardive dyskinesia
Evidence in favour
· vitamin E is of benefit in rodent models of TD
· some studies have shown increased blood or CSF levels of lipid peroxidation
byproducts in patients with TD compared to those without TD
Problems
· Vitamin E treatment of TD does not work

205
Q

GABA insufficiency theory TD

A

Proposed mechanism
Long-term treatment ‡ destruction of GABAergic neurones in the striatum ‡
reduced feedback inhibition ‡ TD
or
Long-term treatment ‡ reduced GABAergic activity in the pars reticulata of the
substantia nigra ‡ reduced inhibition of involuntary movements ‡ TD
Evidence in favour
· antipsychotic-treated dyskinetic monkeys have a decrease in glutamic acid
decarboxylase in the basal ganglia
· patients with TD have been found, on PM, to have a reduced level of in glutamic
acid decarboxylase in the subthalamic nucleus
· GABAergic agonists such as BZDs, baclofen, and gamma-vinyl GABA have
shown promise as therapeutic agents
Problems
· rodent models do not show consistent changes in GABA function with neuroleptic
treatment
· GABA agonists are not yet effective treatments

206
Q

NA dysfunction theory TD

A

Mechanism
· noradrenergic overactivity contributes to the pathphysiology of TD
Evidence in favour
· patients with TD have significantly higher dopamine b-hydroxylase activity
· platelet alpha-2 adrenoceptor binding and CSF noradrenaline have been correlated
with the severity of TD
Problems
· TD cannot be treated with noradrenergic drugs

207
Q

Main EEG patterns and importance I psychiatry

A

Delta

  • slow wave sleep
  • up to 3hz
  • babies
  • pathologically in metabolic encephalopathy, subcritical lesions, hydrocephalus, deep midline lesions

Theta

  • 4-7hz
  • young children
  • pathological in focal subcortical, deep midline, metabolic encephalopathy, some instances of hydrocephalus

Alpha

  • 8-12hz
  • in closing eyes
  • path in coma

Beta

  • active
  • 12-30hz
  • path with benzodiazepines
  • some antidepressants may increase
208
Q

3hz bilateral symmetrical spike Andy wave

A

Absence seziure

209
Q

Focal slow wave

A

Focal lesion

210
Q

Periodic bi or tri phasic sharp waves

A

CJD

211
Q

Low voltage EEG

A

Huntington’s

212
Q

PET use

A

Allows measurements of blood flow, oxygen extraction, blood volume and glucose metabolic rate

Can be used to determine neurotransmitter turnover
Can be used to assess the receptor occupancy by different medications- using raclopride to estimate dopamine receptor occupancy

Useful to diagnose FTD and VaD

213
Q

SPECT

A

Sensitive to pathology with altered blood flow and metabolism such as epilepsy and AV malformations

Can the differential diagnosis of dementia

Can differentiate between dementia and depression of old age

214
Q

BOLD Fmri

A

It can detect metabolic and hemodynamic changes secondary to alterations in neuronal activity

215
Q

Three stages of translation

A

Initiation
Elongation
Conformational change

216
Q

Missense versus nonsense mutations

A

Missense changes the amino acid coded for

No sense results in a stop signal

217
Q

Main methods of genetic studies

A
1. Analysis of segregation to establish mode of transmission
Adoption studies
Parent as proband
Adoptee as proband
Cross fostering design
  1. Twin studies
    Measures concordance rates, similarity of phenotype between twins
    MZ and DZ
  2. Gene mapping
    Positional- identify location
    Functional- selection of candidate genes
  3. Linkage analysis
    Performed in families with more than one affected
    Cosegregation of polymorphic DNA markers with diseases in families
  4. Association studies
    Frequency of marker in populations of patients and healthy controls
  5. Cytogenetic studies
    Cloning disrupted genes